Középiskolai Matematikai és Fizikai Lapok
Informatika rovattal
Kiadja a MATFUND Alapítvány
Már regisztráltál?
Új vendég vagy?

Fórum: GEOMETRIA

  [1]    [2]    [3]    [4]    [5]    [6]    [7]    [8]    [9]    [10]  

Szeretnél hozzászólni? Jelentkezz be.
[1345] HoA2010-01-05 21:15:15

Nem tudom, nem számoltam végig. Ha a [1341]-ben leírtakat saját magad találtad ki, nyilván tudod, miért. Ha mástól vetted át, akkor dolgozz egy kicsit: bizonyítsd vagy cáfold [1343] utolsó képletét.

Előzmény: [1344] Tym0, 2010-01-05 20:26:30
[1344] Tym02010-01-05 20:26:30

Na most megint jól bekavartál. CSak annyit mondj hogy jó az amit az 1341-es hozzászólásomban írtam. Úgy kijön az általam keresett megoldás?

Előzmény: [1343] HoA, 2010-01-05 19:55:45
[1343] HoA2010-01-05 19:55:45

Ismert, hogy a háromszög körülírt körének K középppontját a csúcsokból álló pontrendszer súlypontjaként úgy tudjuk előállítani, hogy a csúcsokat a megfelelő szögek kétszeresének sinusával súlyozzuk. Lásd pl. Reiman István: Geometria és határterületei:

{\bf K} = \frac{{\bf a} sin 2\alpha + {\bf b} sin 2\beta + {\bf c} sin 2 \gamma}{ sin 2\alpha  + sin 2\beta + sin 2 \gamma }

[1341]-ben a1,a2,a3 a (sík)háromszög oldalhosszainak négyzetei, a b1,b2,b3 súlyok a háromszög oldalait hagyományosan a,b,c-vel jelölve az

a2(b2+c2–a2),b2(c2+a2–b2),c2(a2+b2–c2)

mennyiségek. x,y,z a csúcsok ilyen súlyokkal vett súlypontjának koordinátái. Az nem baj, hogy a súlyok összege nem 1, és így a súlypont nincs a háromszög síkjában, mert az utolsó képlettel úgyis a gömbre vetíted. A megoldás akkor helyes, ha be tudod bizonyítani, hogy a súlyok aránya megfelelő, vagyis például

\frac { a^2 ( b^2 + c^2 - a^2 )  }{ b^2( c^2 + a^2 - b^2) } = \frac { sin 2\alpha }{ sin 2\beta }

Előzmény: [1341] Tym0, 2010-01-05 18:27:01
[1342] laci7772010-01-05 19:41:20

Sziasztok, és b.ú.é.k. mindenkinek!

A Geometriai feladatok gyűjteménye I. 2776-os feladata sajnos megfogott. Tudna valaki segíteni benne? A feladat: Adott R sugarú gömbk köré írjunk olyan egyenes körkúpot, hogy térfogatának és a gömb térfogatának aránya adott k legyen. Határozzuk meg a kúp alapkörének a sugarát (r-t).

Addig jutottam, hogy r négyzet*m = 4*R köb*k (azaz gyakorlatilag semeddig), de a körkúp magassága (m), alkotója és sugara kívánatos aránya már kifogott rajtam.

Minden segtséget előre is köszönök! Sziasztok: Laci

[1341] Tym02010-01-05 18:27:01

Ehhez mit szóltok? Vagy ez ugyanaz amit ti mondtatok? Szerintem ez jó lesz. Szerintetek?

A gömb középpontja legyen az origó, a gömb sugara legyen R.

A kiindulási pontok a gömbön legyenek (x1,y1,z1), (x2,y2,z2), (x3,y3,z3).

Sorra számold ki az alábbi mennyiségeket:

a1 := (x2-x3)2 + (y2-y3)2 + (z2-z3)2

a2 := (x3-x1)2 + (y3-y1)2 + (z3-z1)2

a3 := (x1-x2)2 + (y1-y2)2 + (z1-z2)2

b1 := a1*(a2+a3-a1)

b2 := a2*(a3+a1-a2)

b3 := a3*(a1+a2-a3)

x := b1*x1 + b2*x2 + b3*x3

y := b1*y1 + b2*y2 + b3*y3

z := b1*z1 + b2*z2 + b3*z3

c : = R/gyök(x2+y2+z2)

A gömbön a körülírt kör középpontjának keresett koordinátái (c*x,c*y,c*z).

Előzmény: [1340] HoA, 2010-01-05 11:40:36
[1340] HoA2010-01-05 11:40:36

Az eddigiek alapján a lépések:

-Adottak A, B, és C földrajzi koordinátái, északi szélesség = \phi , keleti hosszúság = \alpha

-Átszámítjuk Descartes-koordinátákba : Pz=sin\phi;Px=cos\alpha,Py=sin\alpha ( P = A,B,C )

-Válasszuk úgy a jelölést, hogy ABC pozitív körüljárású \Delta legyen

-Képezzük az N = (B-A) x (C-A) vektorszorzatot, ez a gömb középpontjából kifelé mutat.

-A keresett középpont földrajzi koordonátáit az előzőek alapján kapjuk: sin \phi=Nz/|N| , tg \alpha=Ny/Nx

Előzmény: [1334] Tym0, 2010-01-04 22:31:59
[1339] HoA2010-01-05 11:14:08

„Mindenkinek” igaza van, függetlenül attól, hogy gömbi vagy Descartes koordinátákat használunk.

-a gömb 3 különböző pontja, mint 3 térbeli pont, meghatároz egy S síkot

-ez a sík a gömböt egy körben metszi, és mivel a 3 pont a síkon is és a gömbön is rajta van, ez a kör éppen a 3 pont által meghatározott háromszög körülírt köre

-A BohnerGéza által javasolt vektorszorzat S (egy) N normálvektora, tehát S-re merőleges.

-A gömb középpontjából a gömböt metsző S síkra bocsátott N merőleges S –et a gömb és S metszésvonalát képező kör középpontjában döfi ( szimmetria ) . Talán ez hiányzott a leírtakhoz.

-N a gömböt abban a pontban metszi, amelyik egyenlő távolságra van a 3 adott ponttól – a földgömbnek ebbe a ponjába beszúrt körzővel a 3 ponton áthaladó kört lehet rajzolni

-A körközéppont földrajzi koordinátáinak meghatározásához N hossza lényegtelen. A Descartes koordinátáknak csak itt van szerepe. Ha a földrajzi szélességet \phi-vel, a hosszúságot \alpha–val jelöljük, akkor sin \phi=Nz/|N| , tg \alpha=Ny/Nx

Előzmény: [1338] sakkmath, 2010-01-05 09:59:14
[1338] sakkmath2010-01-05 09:59:14

Szerintem Jonasnak (1328) igaza van akkor, ha a gömbháromszög csúcsai euklideszi koordinátákkal adottak.

Ha viszont az adott koordináták gömbi, földrajzi koordináták, akkor az eddigi hozzászólások nem érvényesek, ugyanis a többi hozzászóló is euklideszi koordinátarendszerben gondolkodott.

Előzmény: [1336] Tym0, 2010-01-05 01:38:08
[1337] Fálesz Mihály2010-01-05 09:59:07

Igaza van Bohner Gézának, egy kicsit túlbonyolítottam. :-)

Előzmény: [1331] Tym0, 2010-01-04 21:09:49
[1336] Tym02010-01-05 01:38:08

Ez mind oké. De foylton síkot említesz. Egy gömfelületen levő háromszög nem lehet sík hiszen a gömbnek a felületén van. Vagy én vagyok a hülye és én nem értem...

Előzmény: [1335] BohnerGéza, 2010-01-04 23:08:22
[1335] BohnerGéza2010-01-04 23:08:22
Előzmény: [1334] Tym0, 2010-01-04 22:31:59
[1334] Tym02010-01-04 22:31:59

A lépéseket próbáld meg leírni lécci. Most ott tartok hogy van 3 (a háromszög csúcspontjai) + 3 (a háromszög oldalainak felezőpontjai) koordinátapontom (amik ugye x,y,z koordináták mert térről beszélünk). És ugye a göm középpontjának koordinátja ami ugye x,y,z alakban 0,0,0. Ezután mi jön? Mik a lépések?

[1333] BohnerGéza2010-01-04 21:25:57

Vektorokkal egyszerűen megy:

Vegyük a gömb kp-jából a kör kp-jába mutató vektort, osszuk a hosszával, szorozzuk a gömb sugarával, majd a gömb kp-jából indítva a keresett pontba mutat.

Előzmény: [1332] BohnerGéza, 2010-01-04 21:14:01
[1332] BohnerGéza2010-01-04 21:14:01

Mivel egyforma húrokhoz egyforma gömbi távolságok tartoznak:

Térben a három ponttól egyenlő távolságra lévő pontok halmaza: a háromszög körülírt körének középpontjában a síkjára állított merőleges. Esesükben ezen rajta van az eredeti gömb középpontja is.

Tehát keressük a körülírt kör kp-ján és a gömb kp-ján átmenő egyenesnek és a gömbnek a megfelelő oldalon lévő metszéspontját.

(Ha nem elég, folytatom.)

Előzmény: [1329] Tym0, 2010-01-04 20:40:33
[1331] Tym02010-01-04 21:09:49

kicsit érthetőbben? Mert ez nekem magas

[1330] Fálesz Mihály2010-01-04 20:43:20

Próbálkozhatsz a három pontra és a gömb középpontjára illeszkedő gömb egyenletével is. (Determináns alakban csak egy pillanat...)

Előzmény: [1327] Tym0, 2010-01-04 17:05:04
[1329] Tym02010-01-04 20:40:33

Dehogy ugyanaz. Mert másképp viselkedik. A gömb az egy térbeli alakzat nem síkbeli és nem euklidészi közegben van vagy valami ilyesmi... Amúgy azon már túl vagyok... És nem lett jó

[1328] jonas2010-01-04 20:26:08

Szerintem számold ki a három csúcs által alkotott síkháromszög köréírt körét, mert az ugyanaz, mint ha gömbháromszögként veszed a köréírt kört.

Előzmény: [1327] Tym0, 2010-01-04 17:05:04
[1327] Tym02010-01-04 17:05:04

Sziasztok!

Egy kis segítséget szeretnék kérni gömbi geometria témakörben!

A problémám a következő:

Kiváncsi vagyok egy gömbháromszög köré írható kör középpontjának koordinátáira, úgy hogy csak a háromszög csúcsainak koordinátái vannak megadva.

Tehát annak a pontnak a koordinátáira, ami a gömbháromszög mindhárom csúcsától egyenlő távolságra van.

Konkrétan: Van három földrajzi koordinátám (századszögmásodperces pontossággal megadva) nem túl nagy távolságra egymástól kb 200km-re. (Mindhárom É.sz. és K.h.) És kiváncsi vagyok annak a pontnak a koordináira, ami mindhárom ponttól egyenlő távolságra van.

Addig már eljutottam hogy a földrajzi koordinátákat átváltottam ekvatoriális, azaz gömbi koordinátákká. És a háromszög mindhárom oldalának felezőpontjai is megvannak. Itt akadtam el...

Arra gondoltam hogy elég valamely két oldal felezőmerőleges gömbi főkörének metszéspontjának koordinátáit kiszámolni. De hogyan??????????????

Ja és vigyázni kell, mert a gömbi főkörök két pontban metszik egymást, azok közül csak az egyik lesz jó mert a másik a gömb átellenes pontján van.

Valaki tudna nekem segíteni????????

[1326] HoA2010-01-03 20:41:42

Mivel kedvenc vesszőparipámat, az egységsugarú körbe írt szabályos 18-szög tulajdonságait érinti, B.4221 elemi megoldását feltettem http://www.komal.hu/forum/forum.cgi?a=to&tid=26&tc=500 -ba ( Lejárt határidejű KÖMAL feladatokról )

[1325] BohnerGéza2009-12-02 22:57:49

HoA! Szép!

Ennek a feladatnak egy sok számolásos megoldásáról hallottam, sajnos nem láttam. Az inverzióval átalakított feladatot azért írtam, hátha sikerül egy, az utolsó mondatodnak megfelelő, megoldás összehozni. (Nem adtam föl.)

Előzmény: [1324] HoA, 2009-12-02 21:15:22
[1324] HoA2009-12-02 21:15:22

Az egység sugarú k körön jellemezzük S helyzetét az ST’T = \phi szöggel. k* sugara legyen r. Az AB ív felezőpntja C, k1 és k2 metszéspontja D, k* középpontja O*, O* és S merőleges vetülete TT’ –re E illetve F, végül k2 és k3 metszéspontja M. A akkor és csak akkor van az MO egyenesen, ha az ATO és ABM derékszögű háromszögek hasonlók, vagyis ha \frac{BM}{TO} = BM = \frac{AB}{AT} . S a k és k* körök hasonlósági középpontja, így O*O=r-1 és CT=(r-1)TS . T’S=2cos\phi, SF=2cos\phisin\phi és így O*E=m=2cos\phisin\phi(r-1) . Legyen az AB húr hossza 2h . \frac {BM}{TT’} = \frac {BM}{2} = \frac{BD}{TD} = \frac{TD + m -h}{TD} = \frac{ 2 tg \phi + m -h}{ 2 tg \phi } , BM = \frac{ 2 tg \phi + m -h}{ tg \phi } Erről kell belátni, hogy megegyezik \frac {AB}{AT}-vel, vagyis \frac {2 h}{h + m}-mel. Felhasználjuk, hogy a szelőtétel értelmében AT.TB=CT.TS , (h+m)(h-m)=2sin\phi.(r-1)2sin\phi=4(r-1)sin2\phi. BM = \frac{ 2 tg \phi + m - h }{ tg \phi } = \frac {2 h}{h + m} , (2tg\phi+m–h)(h+m)=2h.tg\phi=2tg\phi(h+m)(h-m)(h+m)=2h.tg\phi+2mtg\phi-(h-m)(h+m) . 2mtg\phi=(h-m)(h+m) A baloldal 2mtg\phi=4cos\phisin\phi(r-1)tg\phi=4(r-1)sin2\phi , a feltétel teljesül.

Jó lenne egy szemléletesebb megoldás, esetleg az inverzió előtti feladatra is.

Előzmény: [1315] BohnerGéza, 2009-11-24 21:26:53
[1323] HoA2009-11-30 15:29:28

A kör középpontján áthaladó körökkel és egyenesekkel a feladat nagyon inverzió szagú. Megadom az inverzióval keletkező feladatot és ábráját (zöld vonalak) , mert a megoldás így sem triviális.

Jelöljük k-val az O középpontú, az S és T ponton átmenő kört, T’-vel a T-ből induló átmérő másik végét. Legyen k1 k T-beli érintője, k2 az ST' egyenes. Jelöljön k* egy k-t magába foglaló és S-ben érintő kört. k* és k1 metszéspontjai legyenek A és B. Legyen k3 a B-n átmenő TT'-vel párhuzamos egyenes. Bizonyítandó, hogy a k2 és k3 metszéspontján valamint O-n áthaladó egyenes tartalmazza A-t.

Előzmény: [1315] BohnerGéza, 2009-11-24 21:26:53
[1322] BohnerGéza2009-11-27 13:29:45

Egy észrevétel, ami segítheti a megoldást:

Jelölje k2 és k3 O-tól különböző metszéspontja C. Úgy tűnik, hogy ABC szög derékszög, azaz BC párhuzamos k1-k* centrálisával.

Előzmény: [1321] BohnerGéza, 2009-11-27 02:30:00
[1321] BohnerGéza2009-11-27 02:30:00

Köszönöm HoA értelmezését! Igen fáradtan fogalmaztam meg a feladatot, illett volna ábrát is adni.

Nekem mindig pontosan adja az "egyenest" az Euklides.

Előzmény: [1318] HoA, 2009-11-26 12:07:57
[1320] SmallPotato2009-11-26 14:42:53

Jogos ... valóban. A határozott névelő tévesztett meg: "Jelölje k* a k-t belülről S-ben érintő ..." - és egy lehetőségre asszociáltam. Bocsánat.

Előzmény: [1318] HoA, 2009-11-26 12:07:57
[1319] HoA2009-11-26 12:34:11

Erről lenne szó? k2 és k3 egyik metszéspontja nyilván O. A és B felcserélhető ( piros és kék kör illetve egyenes ).

Előzmény: [1318] HoA, 2009-11-26 12:07:57
[1318] HoA2009-11-26 12:07:57

Illetve mégegyszer átolvasva, az "O-t tartalmazó" nyilván úgy értendő, hogy nem a körvonal, hanem a körlap tartalmazza O-t. Elnézést, Géza!

Előzmény: [1317] HoA, 2009-11-26 12:05:38
[1317] HoA2009-11-26 12:05:38

Igen, nekem is ez jött ki. k1 és k* meghatározásában szerepel, hogy O-n áthaladnak.

Előzmény: [1316] SmallPotato, 2009-11-25 17:54:58
[1316] SmallPotato2009-11-25 17:54:58

A szövegezés alapján nekem úgy tűnik, hogy k1 és k* egyaránt a k kört belülről érintő és k-hoz képest feleakkora sugarú kör. De akkor egyik metszéspontjuk O, miáltal a "jelölje ... k* és k1 metszéspontjait A és B" számomra nem igazán jól értelmezhető.

Rosszul értettem valamit?

Előzmény: [1315] BohnerGéza, 2009-11-24 21:26:53
[1315] BohnerGéza2009-11-24 21:26:53

Jelöljük k-val az O középpontú, az S és T ponton átmenő kört, T’-vel a T-ből induló átmérő másik végét. Legyen k1 az OT Thálesz-köre, k2 az S-en, T’-n és O-n átmenő kör. Jelölje k* a k-t belülről S-ben érintő, O-t tartalmazó kört és a k* és k1 metszéspontjait A és B. Már csak a k3-at határozom meg, jelölje a TT’-t O-ban érintő B-n átmenő kört. Bizonyítandó, hogy a k2 és k3 metszéspontjain átmenő egyenes tartalmazza A-t.

[1314] sakkmath2009-11-24 12:09:58

Elnézést, fáradt voltam, elírtam. Helyesen:

"...amikor M-et az AA1 szakasz A-n, illetve A1-en túli meghosszabbításain mozgatjuk."

Előzmény: [1313] sakkmath, 2009-11-23 11:17:38
[1313] sakkmath2009-11-23 11:17:38

Megvizsgálhatók azok az esetek is, amikor M-et a DA1 szakasz D-n, illetve A1-en túli meghosszabbításain mozgatjuk. A Cabri kiírással jelzi, hogy az M által bejárt útvonal egyes csatlakozó szakaszain éppen milyen kúpszelet \Gamma1 és \Gamma2. (Van olyan szakasz is, amikor egy lokális kúpszeletről nem tudja megmondani, hogy az konkrétan melyik, s ez nyilván a program úgynevezett modellhibájával magyarázható.) Érdemes lenne kideríteni, hogy a kiinduló szerkesztéssel milyen kapcsolatban vannak ezek a fázisváltások, melyeknél tehát az egyik kúpszeletfajtából hirtelen egy másikba vált \Gamma1, vagy \Gamma2. Vajon megszerkeszthetők-e az ilyen váltásokhoz tartozó M-ek?

Mindezt nem feladatkitűzésként, hanem egyfajta töprengő lezárásként írtam. Úgy tűnik ugyanis, hogy ez az új kérdéskör – legyen bármennyire ígéretes és izgalmas – túlmutat e FÓRUM jellegén és keretein, és persze az én igencsak szerény ismereteimen :(.

Ismét megköszönöm HoA hozzászólásait, megoldásait. Sokat tanultam belőlük.

Előzmény: [1312] HoA, 2009-11-11 14:59:44
[1312] HoA2009-11-11 14:59:44

3. ábra

Előzmény: [1311] HoA, 2009-11-11 14:59:01
[1311] HoA2009-11-11 14:59:01

2. ábra

Előzmény: [1310] HoA, 2009-11-11 14:58:12
[1310] HoA2009-11-11 14:58:12

M-et DA1-en mozgatva (D az ábrákról lemaradt) azt tapasztaljuk, hogy \Gamma1 és \Gamma2 hiperbola - a hat-hat pont nem konvex sokszöget alkot, a kúpszelet bizonyításnál pedig nem használtuk ki, hogy M a háromszögön belül van. Amíg M D-hez van közel, Q1 az AA1 egyenesnek C-vel, Q2 pedig a B-vel azonos oldalán van. (1.ábra) . Ha M A1-hez van közel, fordított a helyzet (2.ábra). A két esetet az az M0 választja el, amelyre CC1 és A1B1 párhuzamos. (3. ábra). Mivel A1B1B\angle=A1AB\angle=\alpha/2 , váltószöge B1MC\angle is ekkora, CMB\angle=\pi-\alpha/2, M ekkor BC ilyen látószögű körívén van. Ha BC felezőmerőlegese k-t az A1-től különböző A2-ben metszi, M0 éppen az A2 középpontú, A2B sugarú kör és az AA1 egyenes metszéspontja. Ekkor BB1 és A1C1 is párhuzamos, Q1 és Q2 a végesben nem jön létre, hanem annak a hiperbolának a végtelen távoli pontjai, amelyik a P2P5R1R2 pontokon halad át és aszimptotái BB1 és CC1 irányúak.

Ez azonban nem a 158/6. feladat 2. pontjában keresett M0, hiszen a P2 illetve P5-beli érintőkre továbbra is igaz, hogy BC1 ill. CB1 és AA1 metszéspontján haladnak át, márpedig a szemlélet alapján R1 és R2 nincsenek ezen a két érintő egyenesen.

Előzmény: [1308] sakkmath, 2009-10-31 12:25:42
[1309] HoA2009-10-31 17:10:08

Eddig nem ismertem, de sajnos most sem igazán. Oda belépve ugyanis csak egy csomó hirdetés jelent meg - meg egy anchor a www.komal.hu- ra - valamint egy kiírás , hogy "Az Internet Explorer nem tudja megjeleníteni" , de hogy mit, az már nem látszik. Talán valami újabb böngészőt igényel.

Előzmény: [1307] Zsodris, 2009-10-31 10:38:14
[1308] sakkmath2009-10-31 12:25:42
Előzmény: [1306] sakkmath, 2009-10-30 11:57:06
[1307] Zsodris2009-10-31 10:38:14

Sziasztok!

Ismeritek a www.silverpen.eu oldalt?

Szerintem a legjobb ingyenes vektorgrafikus program. Telepíteni sem kell. Ideális geometriai feladatok feladásához, megoldásához.

[1306] sakkmath2009-10-30 11:57:06

Köszönöm a megoldást. Holnap fölteszem a [1293]-ban jelzett kiterjesztést (addig még ellenőriznem kell valamit).

Előzmény: [1305] HoA, 2009-10-26 10:38:11
[1305] HoA2009-10-26 10:38:11

Bár az eddigiekből következik, mivel tételesen még nem szerepelt 158/4/b megoldása, megadom: A hatszög csúcsait R1P2Q2R2P5Q1 sorrendben véve

R1P2\capR2P5=A

P2Q2\capP5Q1=A1

Q2R2\capQ1R1=M

, a három metszéspont egy egyenesen van, így a hat csúcs egy kúpszeleten helyezkedik el. ( Hogy ez ellipszis-e, arra ld. [1299] )

Ezután rátérhetünk 158/4/c –re. P1P2P3P4P5P6 ellipszisének P2-beli érintője legyen t1, ennek P4P6-tal alkotott metszéspontja T . A P2P2P3P4P6P1 ellipszisbe írt „hatszögre”

P2P2(=t1)\capP4P6=T

P2P3\capP6P1=C1

P3P4\capP1P2=B

, T rajta van a BC1 egyenesen. A P2P2P5P4P6P3 hatszögre

P2P2(=t1)\capP4P6=T

P2P5\capP6P3=M

P5P4\capP3P2=A1

, T rajta van az MA1 egyenesen. T tehát BC1 és MA1 metszéspontja, t1 a P2T egyenes.

R1P2Q2Q1P5R2 ellipszisének P2-beli érintője legyen t2, ennek Q1R2-vel alkotott metszéspontja U . A P2P2R1Q1R2Q2 ellipszisbe írt hatszögre

P2P2(=t2)\capQ1R2=U

P2R1\capR2Q2=B

R1Q1\capQ2P2=C1

, U rajta van a BC1 egyenesen. A P2P2P5Q1R2Q2 ellipszisbe írt hatszögre

P2P2(=t2)\capQ1R2=U

P2P5\capR2Q2=M

P5Q1\capQ2P2=A1

, U rajta van az MA1 egyenesen. U tehát BC1 és MA1 metszéspontja, t2 a P2U egyenes. Vagyis T=U és így t1=t2, a két ellipszis P2 -beli érintője közös, érintik egymást. Az ábra szimmetriája miatt P5 -re hasonló bizonyítás adható.

Előzmény: [1293] sakkmath, 2009-10-06 17:56:28
[1304] sakkmath2009-10-26 09:50:51

Egyetértek HoA értékelésével. Most már nekem is úgy tűnik, hogy B.3869-ben nem lehet elemi eszközökkel bebizonyítani a BC-vel nem párhuzamos hatszögfőátlók M-re illeszkedését. Az elmúlt napokban sokat kísérleteztem e témában, de eredménytelenül. Köszönet illeti HoA-t - s talán még valakit :) -, hogy a helyzet tisztázódott.

Előzmény: [1301] HoA, 2009-10-20 16:17:28
[1301] HoA2009-10-20 16:17:28

Sajnos elképzelhetőnek tartom, hogy B.3869 és F.2857 olyan értelemben ikrek, hogy B.3869 –ben , ahol M a szögfelezőn van, valójában azt lehet bizonyítani elemi eszközökkel, hogy a hatszög BC-vel párhuzamos átlója átmegy M-en – és a másik két átlóról nem sikerül, míg F.2857-ben, ahol M az oldalfelező merőlegesen van, nem véletlenül azt kell – és lehet – elemi úton bizonyítani, hogy a hatszög átlói között van két olyan, amelyik M-ben metszi egymást – és az oldalfelezőre merőleges oldallal „párhuzamos” hatszögátlóról nem esik szó.

Előzmény: [1300] sakkmath, 2009-10-14 17:45:24
[1300] sakkmath2009-10-14 17:45:24

Köszönöm HoA újabb megoldásait.

Ha jól értem, a 2)-es kérdés így fejthető ki: Ismerek-e olyan bizonyítást, ami úgy igazolja azt, hogy a Pi hatszög kúpszeletbe írt, hogy közben nem használja fel a főátlók azon tulajdonságát, hogy áthaladnak az M ponton?

A válaszom: nem ismerek ilyen bizonyítást és attól tartok, hogy talán nem is létezik ilyen. Lehetséges viszont, hogy e bizonyítás létezésének eldöntéséhez közelebb vinne, ha valaki elemi úton megoldaná 158/5 ama esetét, amikor M a szögfelezőn van. Ez utóbbi elemi bizonyítás biztosan létezik, hiszen az ikerfeladat F. 2857-re is van elemi bizonyítás (a KöMaL közölt egy ilyet anno)...

Elképzelhető, hogy a vizsgált feladatcsoport egy újabb kiterjesztése is közelebb visz a 2)-es a kérdésben megjelölt bizonyítás létezésének megítéléséhez. (Ezt a kiterjesztést később közölném, a továbbiakban beérkező megoldás(ok) után, ugyanis azokkal is összefügg.)

Előzmény: [1299] HoA, 2009-10-14 11:07:37
[1299] HoA2009-10-14 11:07:37

Azt hiszem nem lövöm le a többi alfeladatra beérkező megoldásokat és nem okozok meglepetést, ha megadom 158/4/a megoldását : A hatszög csúcsait P1P2P5P4P3P6 sorrendben felvéve a „szemközti” oldalak metszéspontjai B,MésB1 , egy egyenesre esnek, így a hat pont egy ellipszisen – vagy legalábbis egy kúpszeleten helyezkedik el.

Kérdéseim: 1) A szemléleten kívül mi igazolja, hogy a kúpszelet ellipszis? – Természetesen nem a görbe egyenletének együtthatóiból képezhető determinánsok vizsgálatára gondolok. 2) Sakkmath tud-e 158/4/a-ra 158/5-öt nem felhasználó megoldást?

Előzmény: [1293] sakkmath, 2009-10-06 17:56:28
[1298] HoA2009-10-12 15:18:42

[1293] TÉTEL-ének - és egyben 158/5nek a bizonyítása: A [1283]-éhoz hasonló ábra csak a könnyebb azonosíthatóság kedvéért. Nem használjuk ki, hogy körülírt körről van szó, tetszőleges kúpszelet lehet, és M-ről sem tesszük fel, hogy a szögfelezőn van. Az ABCA1B1C1 hatszög csúcsainak megfelelő sorrendezésével a Pascal-tétel szerint adódik a TÉTEL állítása. Más sorrendezéssel a másik két átlóról ugyanígy kimutatható, hogy M-en haladnak át.

Előzmény: [1296] sakkmath, 2009-10-09 11:46:36
[1297] sakkmath2009-10-10 19:07:10

Az utolsó bekezdést törlöm. Itt nem használható a Brianchon-tétel.

Előzmény: [1296] sakkmath, 2009-10-09 11:46:36
[1296] sakkmath2009-10-09 11:46:36

Ez a megoldási kísérlet érdekes és visszautal a feladatok gyártástechnológiáját megvilágító egykori "oldalfelező merőleges - szögfelező" cserére.

Ha bebizonyítjuk [1293] TÉTELét (ami voltaképpen - kis bővítéssel - a már említett 158/5. feladat), az egyik lehetséges bizonyításból (Pascal ...) az is kiderülhet, hogy a sejtés erősíthető: az ellipszisen túl, más kúpszeletekre is igaz az állítás.

Most jutott eszembe egy másik, (esetleg) szóba jövő bizonyítási módszer, a Brianchon-os. De ez (ha egyáltalán jó irány) messzire vezet, időigényes, inkább nem részletezem ...

Előzmény: [1295] HoA, 2009-10-07 15:55:37
[1295] HoA2009-10-07 15:55:37

158/4 megoldási kisérletei során merült fel az ötlet: vessük alá az ábrát egy olyan projektivitásnak, mely B-t és C-t helyben hagyja, A-t és M-et viszont BC felező merőlegesére viszi. Ekkor az egyenesek egyenesek maradnak, de a körülírt kör már nem lesz kör. Innen a sejtés: [1293] TÉTEL-e erősíthető: nem kell a körülírt kör, ellipszisre is igaz az állítás.

Előzmény: [1293] sakkmath, 2009-10-06 17:56:28
[1294] HoA2009-10-07 09:52:55

Addig is egy projektív, de rövid megoldás 158/2re: B1P5R2 és C1P2R1 háromszögek megfelelő oldalegyenesei az egy egyenesbe eső A1,A,M pontokban metszik egymást. A Desargues-tétel értelmében ekkor egy pontra nézve is perspektívek. Mivel C1B1 és P2P5 metszéspontja S, R1R2 is itt halad át.

Előzmény: [1293] sakkmath, 2009-10-06 17:56:28
[1293] sakkmath2009-10-06 17:56:28

Egy újabb, korrekt megoldást láttunk HoA-tól.

Azt hiszem, ideje megnevezni a jelenlegi feladatcsokor ősét, a KöMaL 1991. májusi számában megjelent F. 2857. feladatot. Ebből úgy kaptam például a B. 3869 - et, hogy a feladat szövegében az oldalfelező merőlegest egyszerűen kicseréltem szögfelezőre és különböző összefüggéseket vizsgáltam. Első eredményem az e hozzászólás végén (is) szereplő 158/3.' példa volt, amelyet félretéve jutottam el a végül közölt B. 3869 - hez.

Aki elmélyed a nevezett feladatokban, rájöhet, hogy némelyikben fontos szerep jut egy bizonyos hatszög főátlóinak. Érdemes tehát a feladatok általánosítását ezekre koncentrálva megkeresni. Annál is inkább, mert az a sejtésem, hogy HoA "P1MP4 kollinearitására van egy projektív megoldásom" bejelentése pont erre az általánosításra utalhat. Nézzük tehát feladataink eme "burkológörbéjét", melynek projektív megoldása Jolly Joker-ként gyorsan és sok mindent megválaszol (...):

TÉTEL: Adott az ABC háromszög és a belsejében egy M pont. Az AM, BM, CM egyeneseknek a körülírt körrel alkotott második metszéspontjai létrehozzák az A1B1C1 háromszöget, melynek oldalai az ABC háromszög oldalait egy konvex hatszög csúcsaiban metszik. E hatszög főátlói az M pontban metszik egymást.

A 158/4/b. feladat szerkesztésének ígért kiterjesztését később, egy már beérkezett megoldás után célszerű feltennem.

Végül egy másik megoldás a 158/3. feladatra:

Előzmény: [1292] HoA, 2009-10-04 21:26:00
[1292] HoA2009-10-04 21:26:00

A 158/3. feladathoz: [1283] ábrájára is hivatkozva. Legyen ABC \Delta b és c oldalainak aránya k. AA1 és BC metszéspontját jelöljük T-vel. \alpha szögfelezője az a oldalt ilyen arányban osztja, tehát CT=k.BT . ABC és AP2P5 háromszögek hasonlóságából P5M=k.P2M A1P2P5 és A1P3P4 háromszögek hasonlóságából P4T=k.P3T , így CP4=CT–P4T=k(BT–P3T)=k.BP3 . Q1P5M és Q1P4C illetve Q2P2M és Q2P3B hasonló háromszög párokban a hasonlóság aránya megegyezik, Q1 ugyanolyan arányban osztja P4P5 -öt mint Q2 P3P2 -t, a párhuzamos szelők tételének megfordításából Q1Q2 párhuzamos BC -vel.

P1MP4 kollinearitására van egy projektív megoldásom, de talán tud valaki erre is elemit?

158/4b.-re van egy Pascal tételes bizonyításom, ha mást nem érdekel a feladat, felteszem.

Előzmény: [1291] sakkmath, 2009-10-03 20:27:59
[1291] sakkmath2009-10-03 20:27:59

Köszönöm az elegáns megoldást!

Kérdésed után érdemes kitérni olyan további, ki nem mondott, de a [1283]-as ábráról könnyen leolvasható összefüggésekre (sejtésekre) is, melyeket szintén be lehet bizonyítani a projektív geometria alkalmazása nélkül. Egy ilyen a - dinamikus geometriai programok által sugalmazott - következő, 1. sejtés:

A P1P4 és P3P6 szakaszok (hatszögátlók) az M pontban metszik egymást. (Ha ennek igazolását feladatként tűzzük ki, ez a 158./5. feladat lehetne.)

Pár napon belül egy további sejtést is ismertetek, ami a 158/4/b. feladat szerkesztésének kiterjesztése lenne (örülnék, ha ebben valaki megelőzne a vonatkozó megoldásával).

Végül álljon itt egy "minimálábra" a 158./3. feladat megoldásához arra az esere, ha valakit zavarna a [1283]-as rajz zsúfoltsága:

Előzmény: [1288] HoA, 2009-09-30 09:51:33
[1290] sakkmath2009-09-30 12:09:04

... a projektív geometria mellőzésével.

Előzmény: [1289] sakkmath, 2009-09-30 11:39:41
[1289] sakkmath2009-09-30 11:39:41

A 158/3. feladat elemi geometriai módon megoldható.

Előzmény: [1288] HoA, 2009-09-30 09:51:33
[1288] HoA2009-09-30 09:51:33

A 158/1 feladathoz:

Legyen az AP2P5\Delta körülírt köre k1. BC és P2P5 párhuzamossága miatt AP2P5\Delta és ABC\Delta hasonlóak, egymásból A középpontú nagyítással származtathatók, ez körülírt köreikre is áll, ezért k és k1 A-ban érinti egymást, t a két kör hatványvonala. Ossza a CC1 egyenes ABC\Delta \gamma szögét \gamma1=ACC1 és \gamma2=C1CB szögekre. Ekkor C1B1P5\angle=\alpha/2+\gamma2 P2BC1\angle=\gamma1 BC1P2\angle=\alpha/2 AP2C1\angle=\alpha/2+\gamma1 , P5P2C1\angle=\alpha/2+\gamma1+\beta C1B1P5\angle+P5P2C1\angle=180o , B1C1P2P5 húrnégyszög, körülírt köre legyen k2 . B1C1 k és k2 hatványvonala, P2P5 k1 és k2 hatványvonala. A három kör páronként vett hatványvonalai egy pontban metszik egymást (S)

A további feladatokra van valakinek nem projektív geometriai megoldása?

Előzmény: [1283] sakkmath, 2009-09-26 17:52:54
[1287] PuzzleSmile2009-09-28 12:36:22

Nem erről van szó.

Olvassuk össze a következő sor vastagított részét: "C1-ből és L*-ból is béta szögben látszik az AM szakasz".

Tehát: a béta nagyságú látószög hiányzó szárát pótoltam.

Előzmény: [1286] BohnerGéza, 2009-09-27 20:24:37
[1286] BohnerGéza2009-09-27 20:24:37

Jogos! Kösz!

(Az AC1 berajzolása kicsit fölösleges azért! A C'-ből csak egy A jelű pontnak látszik.)

Előzmény: [1285] PuzzleSmile, 2009-09-27 19:34:54
[1285] PuzzleSmile2009-09-27 19:34:54

HoA [1278]-as megjegyzése a joke-ról találó ... :)

HoA [1276]-os kiegészítését elfogadva, az alábbi négy, piros puzzledarabkát helyezem el Bohner Géza [1274]-es megoldásában. Az így korrigált puzzle-t - Géza utólagos engedelmére számítva - idemásolom:

Előzmény: [1275] PuzzleSmile, 2009-09-23 11:05:28
[1284] sakkmath2009-09-27 11:32:04

4/b. feladat: Szerkesszük meg a két ellipszis érintkezési pontjaihoz tartozó érintőit!

(Ez a részfeladat - a szerkesztési eljárást bemutató - bizonyítandó állítás formájában is megfogalmazható. Ez viszont könnyítést jelenthetne, s esetleg elrontanám vele a megoldó(k) örömét ...)

Előzmény: [1283] sakkmath, 2009-09-26 17:52:54
[1283] sakkmath2009-09-26 17:52:54

Egy saját feladatcsokrot

ajánlok a Fórum olvasóinak,

megoldóinak figyelmébe.

158. /1. - 4. feladatok:

Előzmény: [1266] sakkmath, 2009-09-11 16:16:11
[1282] jonas2009-09-25 11:31:06
Előzmény: [1280] PuzzleSmile, 2009-09-25 10:34:31
[1281] HoA2009-09-25 11:28:48

Ezek után elárulhatom, hogy [1270] is egy, a [1274]-hez hasonló elemi geometriai megoldás átalakítása inverzióssá: AMNB1 húrnégyszög, mert MN A-ból és B1-ből is \alpha/2 szög alatt látszik. Így ANM\angle=AB1M\angle=AB1B\angle=ACB\angle. NM és BC párhuzamosak és a vége mint [1270] végén.

A "négy pont egy körön van" egyenértékű azzal, hogy egyiküket az inverzió középpontjának választva a másik három pont képe egy egyenesen van.

Előzmény: [1279] BohnerGéza, 2009-09-25 09:54:02
[1280] PuzzleSmile2009-09-25 10:34:31

A puzzle 4 darabja még hiányzik, az egyikük rajzos. Ha holnap sem lesz, aki kirakja őket, vasárnap ezt megteszem én. (Ezek jelentősége már kisebb.)

A (1276)-os "foltozás" nem inverziós, de az eredeti első bekezdés meghagyásával létezik inverziós befejezés is. Igaz, ez keverék megoldást ad és elromlik a szimmetria.

Előzmény: [1278] HoA, 2009-09-25 06:56:37
[1279] BohnerGéza2009-09-25 09:54:02

Mint írtam:

"Az adott inverzióval játszva sok érdekességet láthatunk, kár, hogy a megoldásnál fölösleges!"

Azaz kár, hogy a megoldásnál fölösleges az inverzió!

Előzmény: [1278] HoA, 2009-09-25 06:56:37
[1278] HoA2009-09-25 06:56:37

Köszönöm PuzzleSmile-nak, hogy ismát ráirányította figyelmemet erre a megoldásra. Azt ugyan még nem árulta el, hogy hol a puzzle, de rájöttem, hogy ha már angolkodunk, akkor ez inkább joke. Ugyanis nem inverziós megoldás. Az első bekezdés helyett nyugodtan írhattuk volna: "Húzzunk párhuzamost M-en át BC-vel, az AB-vel alkotott metszéspont legyen L*. " Ettől persze még a bizonyítás helyes.

Előzmény: [1275] PuzzleSmile, 2009-09-23 11:05:28
[1277] sakkmath2009-09-24 14:05:39

Bohner Gézának és Hoa-nak is köszönöm az érdekes, értékes megoldásokat.

Előzmény: [1274] BohnerGéza, 2009-09-19 23:10:15
[1276] HoA2009-09-23 21:38:59

Ha az ábrát kell szerinted kiegészíteni, áruld el, mire gondolsz. Ha a megoldás szövegét nem találod teljesnek, olvasd el a téma utolsó néhány heti hozzászólásait, melyek alapján az inverzió jópár tulajdonságát már ismertnek vesszük. Azt meg, hogy ML és BC párhuzamosságából következik LN és BC párhuzamossága, úgy értjük, mint [1270] végén: A C1re leírtakat B1re vonatkoztatva kapjuk, hogy MN és BC párhuzamos, tehát L,M,N egy egyenesen vannak és ez párhuzamos BC-vel.

Előzmény: [1275] PuzzleSmile, 2009-09-23 11:05:28
[1275] PuzzleSmile2009-09-23 11:05:28

Hoppá!! ... Ez egy puzzle!

157.feladat: egészítsük ki (1274)-et a hiányzó darabokkal!

Előzmény: [1274] BohnerGéza, 2009-09-19 23:10:15
[1274] BohnerGéza2009-09-19 23:10:15

Legyen az inverzió az az A1 középpontú kör, melyre az A képe M. Ekkor a „k” körülírt kör képe az M-en átmenő BC-vel párhuzamos k’ egyenes. (A1 felezi a BC ívet.) Jelölje L* az AB és k’ metszéspontját.

Mivel C1-ből és L*-ból is béta szögben látszik az AM szakasz, az A, a C1, a L* és az M egy körön van. Ebben a körben a L*M és k-ban az A1C ív is alfa/2 szögben látszik, ezért C1, L* és A1 egy egyenesen van, azaz L* azonos L-lel. Ebből következik, hogy LN átmegy M-en és párhuzamos.

(Az adott inverzióval játszva sok érdekességet láthatunk, kár, hogy a megoldásnál fölösleges!)

Előzmény: [1266] sakkmath, 2009-09-11 16:16:11
[1273] sakkmath2009-09-19 18:37:27

A 9. sor vége helyesen: "\omega és k1 merőlegesen metszik"

Előzmény: [1272] sakkmath, 2009-09-19 18:21:18
[1272] sakkmath2009-09-19 18:21:18

A 2. rész:

Előzmény: [1271] sakkmath, 2009-09-19 18:19:39
[1271] sakkmath2009-09-19 18:19:39

Köszönöm Hoa szép megoldását. Úgy látszik, nincs több hozzászóló, ezért - két részletben - fölteszem saját inverziós levezetésemet, amely különbözik [1270]-től. Az 1. rész:

Az ABC háromszög körülírt köre k, középpontja O, a háromszög A csúcsában lévő szög 2\varphi. Az AA1 szögfelező felezi a 2\varphi szöget, ezért A1AB\angle = CAA1\angle = \varphi, másrészt a BA1 húron nyugvó kerületi szögek egyenlőségéből A1AB\angle = A1B1B\angle =A1CB\angle= \varphi. Az A1C húron nyugvó kerületi szögekre: CAA1\angle = CC1A1\angle = CBA1\angle = \varphi. Látható, hogy az LM szakasz a C1 és az A pontból egyaránt \varphi szög alatt látszik. Ezért az L, M, A, és C1 pontok egy k1 körön sorakoznak, melynek középpontja O1.

Előzmény: [1266] sakkmath, 2009-09-11 16:16:11
[1270] HoA2009-09-15 22:45:37

Legyen az inverzió alapköre az A középpontú, AM sugarú kör. Az inverz pontokat jelöljük vesszőzve. A körülírt kör képe egyenes, ezen jelöljük meg B’,C’,A1’ésB1 pontokat. BC egyenes képe az AB’C’ pontokon át húzott k1 kör, BB1 képe az AB’MB1 pontok k2 köre, A1B1 képe, k3 , AA1’N’B1 pontokon halad át, végül legyen k4 kör az MN egyenes képe az AMN’ pontokon át. BAA1\angle=A1AC\angle=\alpha/2 . B1’A1’A\angle=B1’N’A\angle (k3 –ban közös húr ) . B1’B’A\angle=B1’MA\angle (k2 –ben közös húr ) . Ezért B1’M egyenes az AC és AA1 egyenesekkel AB’A1 höz hasonló háromszöget alkot, ennek külső szöge megegyezik B1’N’A\angle-gel, B1,N’ésM egy egyenesen van. Legyen AB és k4 másik metszéspontja P. N’PA\angle=N’MA\angle=C’B’A\angle. AB’C’ és APN’ hasonló háromszögek, egymásból A középpontú nyújtással keletkeznek. Ez igaz körülírt köreikre is. A tehát k1 és k4 hasonlósági pontja, a két kör érinti egymást. Szimmetrikus módon a C1 -n áthaladó inverz körök vizsgálatával kapjuk, hogy az ML egyenes képe is a k1 kör A középpontú nyújtásával keletkező, M-en áthaladó kör, vagyis k4 ( és P = L’ ) . L,M, N egyazon egyenes pontjai, és mivel k1 -nek és k4 nek nincs A-tól különböző közös pontja, inverz képeik, a BC és LN egyenes párhuzamosak.

Előzmény: [1266] sakkmath, 2009-09-11 16:16:11
[1269] sakkmath2009-09-14 12:21:57

Az archaikus szövegből kihámozható, hogy az Euler-egyenessel kapcsolatos ismert tételről van szó. Olvassuk el itt Dr. Darvasi Gyula: Egy feladat - többféle megoldás című doktori értekezésének 1.5 fejezetét (a 83. oldalon kezdődik, Egy oldallal párhuzamos Euler-egyenes címmel).

Remélem, jó a következtetésem. Az egyes kifejezések modernizálását meghagyom a következő hozzászólónak :)

Előzmény: [1268] jeneit92, 2009-09-12 08:46:28
[1268] jeneit922009-09-12 08:46:28

Sziasztok,találtam egy nagyon érdekes feladatot,ami szerintem a geometriához kapcsolódik: Az tiszta tudékosságban járatos Euler professor Urunk nevezetes léniájárúl Lészen ollybá egy háromszeglemény , melliknek is nehézkedési czentrálisán s ortogonális czentrálisán is által visitáló léniája paralell vala egyvalamely gyepüléniával.Igazoltassák,hogy emez gyepülénia kenyekinek kebeljeinek szorzamányát pótkebeljeinek szorzmányával hányadékul véve mindenkoron 3 adatik.Vajon igaz vala-é az fentebb forgandó theoria visszásítása?" (Gerőcs László Tanár Úr: XVII. századi matekóra című előadásának egy feladata alapján.

Ha bárkinek van ötlete,javaslata,megoldása örömmel veszem,köszi előre is

[1267] BohnerGéza2009-09-11 20:14:40

Ábra a 153. feladat megoldására:

Előzmény: [1247] BohnerGéza, 2009-08-11 05:17:36
[1266] sakkmath2009-09-11 16:16:11

Az elmúlt hónap hozzászólásainak gyakori témája a körre vonatkozó inverzió. Oldjuk meg inverzióval a KöMaL 2005 decemberi számának következő feladatát:

B. 3869. Az ABC hegyesszögű háromszög belsejében, az A csúcsból induló szögfelezőn felvettük az M pontot. Az AM, BM, CM egyeneseknek a körülírt körrel való második metszéspontja rendre A1, B1 és C1. Az AB és a C1 A1 egyenesek az L pontban, az AC és a B1 A1 egyenesek az N pontban metszik egymást. Bizonyítsuk be, hogy az LN szakasz párhuzamos BC-vel.

(A Lap nem közölt inverziós megoldást.)

[1265] HoA2009-09-08 09:34:39

BohnerGéza utólagos engedelmével legyen ez a 156. feladat . Vegyük észre, hogy ez a 154/b feladat általánosításának tekinthető. C-nél derékszögű ABC háromszögre megegyezik a [1260]-ban kitűzött és megoldott 154/b feladattal – pontosabban annak megfordítottjával. Ha ABC C csúcsú egyenlőszárú háromszög, akkor a feladatban szereplő alakzatok szimmetrikusak a C-nél lévő szög felezőjére, a megoldás evidens. Legyen tehát AC < BC.

Először legyen ABC C-nél tompaszögű – a hegyesszögű esetet várom a további érdeklődőktől. C ekkor AB kT Thálesz-körének belső pontja. Az A középpontú C-n átmenő kA kör és kT metszéspontja P, a B középpontú C-n átmenő kB kör és kT metszéspontja Q. A PQ egyenes kA-t még R-ben, kB-t még S-ben metszi. Legyen PR felezőpontja U, QS felezőpontja V. ABQP húrnégyszög. Az ábrán kékkel ill. pirossal jelölt szögeket tartalmazó derékszögű háromszögek hasonlóságából:

PR = 2 \cdot PU = 2 \cdot \frac {PU}{AP} \cdot \frac {AP}{AB}\cdot AB = 2 \cdot \frac {BQ}{AB} \cdot  \frac {QV}{BQ} \cdot  AB = 2 \cdot QV = QS

. Így PQ T felezőpontjának kA-ra és kB-re vonatkozó hatványa megegyezik, T rajta van kA és kB hatványvonalán, a C-n áthaladó AB-re merőleges h egyenesen . P merőleges vetülete AB-n B’, Q-é A’ . A’B’PQ derékszögű trapéz, TC a középvonala, A’B’ felező merőlegese. A’ és B’ egyenlő távolságra van C-től. Ha valakinek nem világos, hogy A’ és B’ a feladatban szereplő inverz képek, vessen egy pillantást [1258] ábrájára.

Előzmény: [1263] BohnerGéza, 2009-09-05 01:06:08
[1264] BohnerGéza2009-09-05 19:38:50

Ez az eredetileg az [1246]-ban kitűzött, majd részleteiben belátott feladatnak egy letisztult bizonyítása a beírt körre. Javaslom a végiggondolását a hozzáírt kör esetére!

154. feladat: Az ABC háromszögben vegyük az A-hoz kapcsolható két érintőkör egyikét - vagy a beírt kört, vagy az A-val szemközti hozzáírt kört - és annak középpontját. Igazoljuk, hogy az ezen középponton átmenő A középpontú alapkörre vonatkozó inverziónál az ABC körülírt körének képe érinti az érintőkört!

Előzmény: [1252] BohnerGéza, 2009-08-13 13:55:33
[1263] BohnerGéza2009-09-05 01:06:08

Tetszőleges A, B és C esetén legyen A' a A-nak inverz képe a B középpontú C-n átmenő körre, B' a B-nek inverz képe az A középpontú C-n átmenő körre.

Bizonyítandó, hogy CA'=CB'.

[1262] BohnerGéza2009-08-27 17:46:46

Mivel a négy háromszög tétel, amire HoA [1260]-ban utal, nem ismert, bár a Fórumban már szerepelt:

Ha négy egyenes négy háromszöget határoz meg, akkor ezek körülírt körei egy ponton mennek át és magasságpontjaik egy egyenesen vannak.

(Azt most nem gondoltam át, hogy ennek használata egyszerűsítheti-e az [1252]-ben írtakat.)

Érdekes következménye ennek: Ha adott egy parabola négy érintője, akkor mivel ezek négy háromszöget határoznak meg, a fókuszt és a vezéregyenest kapjuk.

Előzmény: [1260] HoA, 2009-08-27 14:53:48
[1261] BohnerGéza2009-08-27 17:35:04

HoA [1261] jogos felvetései alapján írom.

Az IA és ID az [1252]-es hozzászólásban meghatározott inverziók. A 154. feladat és vázlatos megoldása is ott szerepel, annak a kiegészítése ez. Belátjuk, hogy O a két alapkör metszéspontja.

A D’-n átmenő AD-re merőleges m egyenes IA-nál és ID-nél is az AD Thálesz-köre, ezért ennek a körnek és m-nek a metszéspontja mindkét inverzió alapkörén van.

Jelölje P az AO egyenes és az ABC kör (nem A) metszéspontját. Bizonyítható középponti-kerületi szögekkel, hogy P az OBC kör középpontja. A DO érinti ezt a kört: DO négyzete = DC*DB = DD’*DA, a befogó-tételből D’O merőleges AD-re.

Ezekből következik, hogy O a két alapkör metszéspontja.

Előzmény: [1260] HoA, 2009-08-27 14:53:48
[1260] HoA2009-08-27 14:53:48

Lassan bár, de haladunk a 151. feladat inverziót használó megoldásának teljes megadása felé. A 154. feladatra történő hivatkozást egyelőre fogadjuk el – később visszatérünk rá. AO Thálesz-körének bevetése nagyon tetszik. A 155. feladat megoldását alkalmazva készen is vagyunk. Mivel ez utóbbi eddig nem szerepelt, álljon itt egy elemi megoldás:

OA’ * OA = OB’ *OB, így OAB és OA’B’ háromszögek hasonlóak, OB’A’\angle=\alpha/2 . Hasonlóan OC’A’\angle=\alpha/2. [1258] alapján OB’D\angle=OC’D\angle=90o , A’B’D\angle=A’C’D\angle , A’B’D és A’C’D körben a közös A’D húrhoz ugyanakkora kerületi szög tartozik, a két kör sugara egyenlő.

És most vissza a 154. feladatra. [1252] valóban hagyott gondolkodnivalót. A 154/a feladathoz nézzük [1253] ábráját. Legyen BAC\angle=\alpha , B’DB\angle=\delta , ekkor a BCC’B’ négyszögben AO és DO merőlegessége miatt B-nél 90o-\alpha/2-\delta/2 , C’-nél 90o+\alpha/2+\delta/2 szög van, BCC’B’ húrnégyszög.

Az A illetve D középpontú megfelelő inverzió léte világos.

A következő bekezdés is teljes odáig, hogy – kis javítással - DB*DC=DB’*DC’=DA*DD’ . Az viszont, hogy ezért D’ = A’, nem adódik közvetlenül, hiszen az IA-ról leírtakat ID-re alkalmazva csak azt kapjuk, hogy A’ rajta van a DBB’ körön. Ahhoz, hogy belássuk, a DBB’ kör ugyanabban a pontban metszi AD-t mint az ABC kör, vagy hivatkoznunk kell a négy háromszög tételére ( DBB’ áthalad ABC és AB’C’ körök metszéspontján ) , vagy kitűzhetjük és megoldhatjuk a 154/c feladatot:

Ha a BCC’B’ húrnégyszög BB’ és CC’ oldalegyenesei A-ban, BC és B’C’ oldalegyenesei D-ben metszik egymást, akkor ABC és DB’B háromszögek körülírt köreinek ( B-től különböző ) M metszéspontja az AD egyenesen van.

Elfogadva tehát, hogy A’ = D’, tűzzük ki általánosan és oldjuk meg a 154/b feladatot: Adott az O1 középpontú I1 és az O2 középpontú I2 inverzió. Bizonyítsuk be, hogy ha I1-nél O2 képe megegyezik I2-nél O1 képével (P), akkor I1 és I2 alapköre merőleges.

Bizonyítás: Tekintsük O1O2 Thálesz-körének és az O1O2-re P-ben emelt merőlegesnek (egyik) Q metszéspontját. O1PQésO1QO2 derékszögű háromszögek hasonlóságából O1P.O1O2=O1Q2 , O1Q tehát I1 alapkörének sugara, Q rajta van I1 alapkörén. Hasonlóan adódik, hogy Q rajta van I2 alapkörén is. A Thálesz-kör miatt a Q-ból az alapkörök középpontjaiba húzott sugarak merőlegesek, így a két alapkör is merőleges.

Visszatérve az eredeti feladatra, a következő állítás „Ez viszont csak úgy lehet, ha az alapkörök átmennek O-n” közvetlenül nem adódik az előzőekből, csak az, hogy az alapkörök átmennek az inverzió középpontok Thálesz-körének egy közös pontján. Így kitűzhető a – 151-et közvetlenül nem támogató –

154/d feladat: Biz. : A 154/c feladat M pontjában az AD-re emelt merőleges áthalad a BCC’B’ húrnégyszög körülírt körének középpontján,

valamint a 151 megoldását előrevivő 154/e feladat: Ha a 154/c feladatban BCC’B’ egyúttal érintőnégyszög is, akkor az M pontban az AD-re emelt merőleges áthalad a BCC’B’ négyszög beírt körének középpontján .

Annak reményében, hogy lesz hozzászóló, majd innen folytatom.

Előzmény: [1259] BohnerGéza, 2009-08-18 20:47:01
[1259] BohnerGéza2009-08-18 20:47:01

A 151. feladat megoldása:

A beírt kör (, vagy az A-val szemközti hozzáírt kör)középpontja O. (Utóbbi esetben az ábrán B és B’ illetve C és C’ szerepet cserél.)

A 154. feladat alapján az A középpontú O-n átmenő körrel adott inverziónál a B’C’ egyenes képe a körülírt (röviden ABC-) kör. Mivel B’C’ érinti az beírt kört, ennek képe lesz az oldalakat és az ABC-kört érintő kör.

Az AO Thálesz-körének a képe az EG egyenes, ezért az EG szakasz felezőpontja O.

A DB egyenes képe az AD’B’-kör, a DC-é az AD’C’-kör és a DO-é az AD’O-kör. Alkalmazzuk az AB’C’ háromszögre a 155. feladatot! Ennek hozzáírt köre az eredeti beírt kör, ez az újabb inverzió alapköre.

Ebben az A AD’B’-kör A’D”B” és az AD’C’-kör A’D”C” képe egybevágó körök és az AD’O-kör képe a A’D” egyenes, a két egyforma kör közös húregyenese, tehát szögfelezője. Az inverzió szögtartása miatt DO szögfelezője a BDC szögnek.

Előzmény: [1217] MTM, 2009-05-12 18:06:51
[1258] BohnerGéza2009-08-15 13:42:59

Ha valaki még most ismerkedne az inverzióval, a 155. feladatot megoldhatja!

Elég hozzá az ábrán látható tulajdonságát ismerni. A k az inverzió alapköre, I a középpontja. A külső P képét a belőle húzott érintők érintési pontjai közti középpont adja, és fordítva: IP'-re P-ben merőleges, érintési pontban érintő, azokból P. (Bizonyítás mehet befogótétellel: IP*IP'=r*r)

Előzmény: [1255] BohnerGéza, 2009-08-14 03:28:49
[1257] BohnerGéza2009-08-15 03:51:51

A 152/3 feladat megoldásához:

Ha két háromszög hasonló és körüljárásuk megegyezik, akkor van olyan forgatva nyújtás, mellyel egyikből a másikat kapjuk. Ez azt is jelenti, hogy ennek fixpontjából bármely pont és képe a forgatás szögében látszik.

Tegyük föl, hogy ABC hasonló A’B’C’-höz. (ábra) Ekkor van olyan pont melyből AA’, BB’ és CC’ egyforma szögben látszik. Erre egy megoldás M. Kérdés lehet e másik ilyen pont, természetesen más szögű forgatással?

Ha azt kapjuk, hogy csak M lehet jó, akkor M csak úgy lehet a középpontos hasonlóság fixpontja is, ha hozzá képest A, B és C azonos fázisban van! Ekkor csak HoA megoldása létezhet.

Még több pontban hiányos!

Előzmény: [1232] HoA, 2009-05-27 13:38:52
[1256] HoA2009-08-14 08:28:18

A 151. feladathoz

Igazoljuk, hogy a BDC szög felezője átmegy a beírt kör középpontján! Emeljük ki [1244] ábrájából a DE’G’\Delta-et. fb a BG’ , fc a CE’ egyenes, ezek metszéspontja a beírt kör középpontja. Alkalmazzuk a Ceva tételnek azt a változatát, hogy a \Delta csúcsain át a \Delta belsejében haladó egyenesek akkor és csak akkor mennek át egy közös ponton, ha az egyeneseknek az oldalakkal bezárt szögeire teljesül

 \frac {sin \alpha_1}{ sin \alpha_2} \cdot  \frac {sin \beta_1}{ sin \beta_2} \cdot  \frac {sin \gamma_1}{ sin \gamma_2} = 1

. A DE’G’\Delta-re tehát [1244] jelöléseivel  \frac {sin \gamma/2}{ sin \beta/2} \cdot  \frac {sin \alpha_2}{ sin \beta/2} \cdot  \frac {sin \gamma/2}{ sin \alpha_1} = 1 -et kell igazolni. Ez viszont éppen [1244] (1) képlete, tehát a feltétel teljesül.

Előzmény: [1254] HoA, 2009-08-13 16:42:35
[1255] BohnerGéza2009-08-14 03:28:49

155. feladat: Legyen az ABC háromszög A-val szemközti hozzáírt köre az inverzió alapköre, mely a BC oldalt D-ben érinti. Bizonyítandó, hogy az ABD kör és az ACD kör képe egybevágó. (Beírt körre is igaz?)

Használható a 151. feladathoz.

[1254] HoA2009-08-13 16:42:35

A 154 ill. 151. feladathoz

Felteszem amire jutottam, bár látom, közben BohnerGéza beírt egy szebb megoldást.

Az A kp-ú t sugarú alapkörre vonatkozó inverziónál a körülírt kör képe egyenes, C és B pontok C’ és B’ képére AB’ / AC’ = AC / AB , így AB’C’ \Delta hasonló ABC-hez. Legyen az arányossági tényező k. AC’ = kc, AB’ = kb , B’C’ = ka . k-ra teljesül, hogy AC’.AC=k.c.b=t2 . CC’B’B akkor lesz érintőnégyszög, ha k ( a + b + c ) = 2 (s-a) . Elegendő tehát azt igazolni, hogy

k = \frac {t^2}{bc} = \frac {s-a}{ s}

t^2 = \frac {\rho^2}{ sin^2 \alpha/2}  = \frac {( s-a ) ( s-b )  ( s-c ) }{ s . sin^2 \alpha/2 } Ezt helyettesítve és (s-a) /s –sel egyszerűsítve, majd a cosinus tétellel

bcsin2\alpha/2=(s-b)(s-c)={a-(b-c)}{a+(b–c)}/4=(a2(b-c)2)/4

2.b.c(1-cos\alpha)=b2+c22.b.c.cos\alpha(b2+c22bc)

Ez pedig valóban azonosság. Így ebben az inverzióban kt a beírt kör inverze, AE / t = t / AE’ , AE’F és AFE hasonló derékszögű háromszögek, AEF és AGF egybevágó derékszögű háromszögek EF = GF és egy egyenesbe esnek, a beírt kör F középpontja tehát EG felezőpontja. Most már csak azt kell igazolni, hogy a BDC szög felezője átmegy a beírt kör középpontján.

Előzmény: [1246] BohnerGéza, 2009-08-11 04:03:12
[1253] BohnerGéza2009-08-13 13:57:44

Ábra a 154. feladat megoldásához.

Előzmény: [1252] BohnerGéza, 2009-08-13 13:55:33
[1252] BohnerGéza2009-08-13 13:55:33

154. feladat:

Az ABC háromszögben vegyük az A-hoz kapcsolható két érintőkör egyikét - vagy a beírt kört, vagy az A-val szemközti hozzáírt kört - és annak középpontját. Igazoljuk, hogy az ezen középponton átmenő A középpontú alapkörre vonatkozó inverziónál az ABC körülírt körének képe érinti az érintőkört!

A 154. feladat megoldása: (VÁZLAT)

Az O középpontú k körhöz az A, valamint az AO-ra O-ban állított merőlegesen lévő D külső pontból húzzunk érintőket k-hoz. Ezek metszéspontjait az A-ból húzott érintőkön jelölje B, B’, C és C’ az A,B’,B illetve A,C’,C sorrendben. (feladatuk szempontjából föltehető, mind létezik)

Először lássuk be, hogy B’BCC’ húrnégyszög! Használjuk ki, hogy A-nál AO és D-nél DO szögfelező, és O-nál derékszög van! 154/a feladat

Ebből következik, van olyan A illetve D középpontú IA illetve ID inverzió, melyekben B-B’, C-C’ illetve B-C és B’-C’ egymás képei.

Az IA-nál a BCD egyenesből a B’C’D’A kör lesz, ahol tehát D’ az AD egyenes és a B’C’A kör metszéspontja. De D’ szükségképpen az ABC körön van, hiszen DB*DC=DB’*DC’=DA*AD’. Ezért fordítva, ID-nél A’ ugyan ez a pont lesz! (A’=D’!)

Azaz az egyik inverziónál a másik középpontjának a képe ugyan az, mint fordítva. Bizonyítandó, hogy a két inverzió alapköre merőleges.154/b feladat

Ez viszont csakúgy lehet, ha az alapkörök átmennek O-n.

Ezzel az ABC háromszög esetén beírt körre, az AB’C’ esetén hozzáírt körre beláttuk a feladatot.

(Remélem, hagytam gondolkodni valót!)

Oldjuk meg a 154. segítségével a 151. feladatot!

Előzmény: [1246] BohnerGéza, 2009-08-11 04:03:12
[1251] BohnerGéza2009-08-12 23:59:15

A 154. feladat megoldásához, ha jól látom, fölhasználható ez az ismert tétel:

A csúcsból induló szögfelező felezi a csúcsból induló magasságvonal és a csúcsot a körülírt kör középpontjával összekötő egyenes szögét.

Előzmény: [1246] BohnerGéza, 2009-08-11 04:03:12
[1250] BohnerGéza2009-08-11 12:34:07

Az ABC háromszög beírt, vagy az A-val szemközti hozzáírt körét értem az A-hoz kapcsolható érintőkörnek. (Bocs, itt valóban úgy is érthető, ahogy az ábrádon szerepel!)

Előzmény: [1249] HoA, 2009-08-11 08:11:04
[1249] HoA2009-08-11 08:11:04

Valamit félreértek. Ugye nem erre az ábrára gondolsz?

Előzmény: [1246] BohnerGéza, 2009-08-11 04:03:12
[1248] HoA2009-08-11 07:33:43

Jogos! 151 kitűzésében nem szerepelt, hogy kt belülről érinti k-t.

Előzmény: [1245] BohnerGéza, 2009-08-11 00:39:33
[1247] BohnerGéza2009-08-11 05:17:36

A 153. feladat megoldása: Tükrözzük B-t a PQ felezőmerőlegesére: B' (ha már ez egy lényeges vonal!).

Bizonyítsuk be, hogy ABB'C húrnégyszög.

Előzmény: [1239] Fálesz Mihály, 2009-07-18 06:28:32
[1246] BohnerGéza2009-08-11 04:03:12

154. feladat: (Ötlet a 151. feladat megoldásához:)

Vegyük az A-hoz kapcsolható két érintőkör egyikét és annak középpontját. Igazoljuk, hogy az ezen középponton átmenő A kp-ú alapkörre vonatkozó inverziónál az ABC körülírt körének képe érinti az érintőkört!

Előzmény: [1217] MTM, 2009-05-12 18:06:51
[1245] BohnerGéza2009-08-11 00:39:33

Megjegyzés a 151/2 feladathoz: Bizonyítsuk be, hogy F az ABC háromszög beírt körének, vagy az A-val szemközti hozzáírt körének középpontja!

(érdemben nem néztem, csak szerkesztőprogrammal)

Előzmény: [1244] HoA, 2009-08-10 23:29:05
[1244] HoA2009-08-10 23:29:05

A 151. feladathoz: Legyen a körülírt kör k, a D, E, G pontokon átmenő kör kt . D a két kör hasonlósági pontja . Ebben a hasonlóságban E, A, G megfelelője rendre E’, A’, G’ . E’A’ párhuzamos az EA egyenessel és E’-ben érinti k-t, ezért E’ az AB ív felezőpontja, E’DA’\angle=EDA\angle=\gamma/2 . Hasonlóan G’ az AC ív felezőpontja, G’DA’\angle=GDA\angle=\beta/2. Ossza a DA egyenes ABC\Delta\alpha szögét BAD=\alpha1 és DAC=\alpha2 szögekre. AG és A’G’ egyenesek távolsága AT2=r(1–cos\beta)=AA’sin\alpha2 , hasonlóan AT1=r(1–cos\gamma)=AA’sin\alpha1 . Innen

\frac {sin \alpha_1}{sin \alpha_2} = \frac {1- cos \gamma }{ 1 - cos \beta } = \frac {sin^2 \gamma/2 }{ sin^2 \beta/2} (1)

AG = AE = t jelöléssel az ADG\Deltaben t.sin\alpha2=DGsin\beta/2 , DG=t.sin\alpha2/sin\beta/2 . AED\Deltaben ugyanígy DE=t.sin\alpha1/sin\gamma/2 . BDC szög felezője messe GE-t F-ben, k-t H-ban. A felezés miatt HDE’=FDE\angle=\beta/2 és HDG’=FDG\angle=\gamma/2 . DEF és DFG \Delta-ek kétszeres területére :

2 T_{DFE} = DE \cdot  DF \cdot sin \beta/2 = DF \cdot t \cdot sin \alpha_1 \cdot \frac {sin \beta/2 }{ sin \gamma/2}

,

2 T_{DGF} = DG \cdot  DF \cdot  sin \gamma/2 = DF \cdot t \cdot sin \alpha_2  \cdot \frac { sin \gamma/2 }{ sin \beta/2}

A területek aránya (1) miatt \frac {T_{DFE}}{T_{DGF}} = \frac {sin \alpha_1}{ sin \alpha_2} \cdot \frac { sin^2 \beta/2}{  sin^2 \gamma/2} = 1 . A két \Delta közös oldalegyenese GE, közös magasságuk a D-ből erre bocsátott merőleges, területük egyenlőségéből következik, hogy GF = FE.

151/2 feladat: Bizonyítsuk be, hogy F az ABC\Delta beírt körének középpontja

Előzmény: [1217] MTM, 2009-05-12 18:06:51
[1243] BohnerGéza2009-08-05 15:44:20

Csak szerkesztőprogrammal játszva: kiderült nem igaz a fordított irány ...

Előzmény: [1242] BohnerGéza, 2009-08-05 14:28:22
[1242] BohnerGéza2009-08-05 14:28:22

Már picit gondolkodva is, nem csak szerkesztőprogrammal játszva:

Ha a nagy gamma (KLM) kör érinti a PQ-t, akkor azt az M felezőpontban teszi. Ekkor a KLM-kör R kp-ja egyenlő távol van P-től és Q-tól. Ha a feladat állítása igaz, M, R és O egy egyenesen van.

Ha ez mindig igaz, abból következik a feladat állítása.

Bizonyítsuk, hogy M, R és O egy egyenesen van!

Előzmény: [1239] Fálesz Mihály, 2009-07-18 06:28:32
[1241] HoA2009-08-05 08:09:20

A 151. feladathoz: Szerkesztőprogrammal nézegetve úgy tűnik, hogy az EG szakasz F felezőpontja ABC háromszög beírt körének középpontja. Talán segít, ha először ezt bizonyítjuk.

Előzmény: [1217] MTM, 2009-05-12 18:06:51
[1240] BohnerGéza2009-08-03 22:50:28

Csak szerkesztőprogrammal játszadozva azt valószínűsítem, hogy P-nek és Q-nak nem kell belső pontnak lennie. (Néha egyszerűbb az általánosabb, itt vszleg mindegy.)

Előzmény: [1239] Fálesz Mihály, 2009-07-18 06:28:32
[1239] Fálesz Mihály2009-07-18 06:28:32

153. feladat (az idei matematikai diákolimpia 2. feladata).

Az ABC háromszög köré írt kör középpontja O. P és Q belső pontjai a CA, illetve AB oldalaknak. Legyen K, L és M a BP, CQ, illetve PQ szakaszok felezőpontja, és legyen \Gamma a K,L,M pontokon átmenő kör. Bizonyítsuk be, hogy ha \Gamma érinti a PQ egyenest, akkor OP=OQ.

[1238] sakkmath2009-06-15 15:55:56

A 2. rész:

Előzmény: [1237] sakkmath, 2009-06-15 15:54:30
[1237] sakkmath2009-06-15 15:54:30

Hasznos volt egy kicsit kibontani a megoldás menetét, több szempontból is. Nekem például ezzel vált világossá, hogy valóban kulcsszerep jut az M pontnak. Melyről most már elárulhatjuk: ez a Miquel-pont, amely nemrég több itteni hozzászólás témája volt.

Múlik az idő, úgy tűnik, ideje feltenni a 152. feladatra egy olyan megoldást, amely a [1232]-ben közölt megoldásod a) megjegyzésére válaszol. Az ugyanitt javasolt 152/3. feladatra még megoldó kerestetik... . Később egyébként megnevezem a feladat elődjét, a megoldás elérhetőségét is megadva.

Következzék tehát a [1230]/152. példa egy újabb megoldása. Az 1. rész:

Előzmény: [1236] HoA, 2009-06-08 11:50:35
[1236] HoA2009-06-08 11:50:35

A leírtakkal egyetértek, sőt nagy részét evidensnek tartom. Megjegyzésem nem kívánt a kitűzés kritikája lenni. Ha már úgyis egy kicsit OFF vagyunk, leírom, hogyan jöttem rá én a megoldásra. Egyrészt hogy bemutassam, milyen értelemben „válik az M pont a megoldás kulcsává”, másrészt mert egy matematika oktatáshoz kapcsolódó fórumon talán nem baj, ha időnként ilyesmit is leírunk.

A három, egymást egy közös pontban metsző kör ábráját nézve feltünt, hogy az ábra szimmetrikus abban az értelemben, hogy egyik körnek sincs kitüntetett szerepe bármelyik másikkal szemben. Logikusnak véltem, hogy a megoldás is szimmetrikus: Ha a három körön futó pont valamelyike M-be kerül, akkor a másik kettő is. Ekkor persze vándorló háromszögünk elfajuló, egy pontra zsugorodik, alakjáról semmit sem mondhatunk – lásd NAGY BUMM hasonlatodat. De ha a futópontok közös M-beli helyzete megfelelő, akkor a feladat szabálya szerint a futópontok 180o-os elforgatottja, vagyis a - később indokolt módon - PaM,PbM,PcM -mel jelölt háromszög is megfelelő. Megrajzoltam PaM,PbM,PcM-et és úgy láttam, hogy oldalegyenesei áthaladnak az Mxy pontokon. Bebizonyítottam, hogy erre a háromszögre ez valóban igaz, és itt nekem is beugrott, valamelyik régi feladatban már szerepelt ez az ábra. Milyen jó lenne, ha a háromszögek többi helyzetében is az oldalegyenesek áthaladnának az Mxy pontokon! Beláttam, hogy tetszőleges ilyen Pa,Pb,Pc háromszög hasonló PaM,PbM,PcM-hez. Innen kezdtem el leírni a megoldást. A futópontok kezdeti felvétele tehát nem „Deus ex machina” módon, hanem az eddig leírtak alapján történt. Most már csak azt kellett igazolni, hogy az Mxy pontokra ileszkedő oldalegyenesekkel rendelkező háromszögek létrejönnek a futópontok kitűzésben leírt mozgása során.

Előzmény: [1235] sakkmath, 2009-05-29 13:50:15
[1235] sakkmath2009-05-29 13:50:15

Örülök, hogy föltetted ezt a szép, a lényegre koncentráló megoldást! Köszönet érte. A feladatnak vannak egyéb, részben általad is jelzett, kidolgozásra érdemes részei, ezért most a magam részéről nem teszek ezeket érintő kiegészítéseket. Abban bízom, hogy valaki(k)nek lehetnek még saját eredményei(k) és 1-2 héten belül esetleg közli(k) azokat. Reagálnék viszont a kitűzési szöveggel kapcsolatos néhány megjegyzésedre, az estleges félreértések elkerülése végett. (Elnézést a terjedelemért.)

Mégegyszer: köszönöm és grat.

Előzmény: [1232] HoA, 2009-05-27 13:38:52
[1234] m2mm2009-05-28 21:32:38

Mivel a kör(ök) érinti(k) a tengelyeket, és (9,-2) pont x koordinátája pozitív, y koordinátája negatív, ezért a kör középpontjának x és y koordinátája ellentettei egymásnak, x pozitív, y negatív. A kör sugarának a hossza a középpont x koordinátájának nagysága, hiszen a kör érinti x-szet. A kör áthalad (9,-2) ponton, tehát: (9-r)2+(-2+r)2=r2, ahonnan r2-22r+85=0, amiből r értékére 5 illetve 17 adódik. Tehát a két kör egyenlete: (x-5)2+(y+5)2=25 illetve (x-17)2+(y+17)2=289.

Előzmény: [1233] Luc, 2009-05-28 21:00:02
[1233] Luc2009-05-28 21:00:02

Sziasztok! Problémám akadt egy koordináta geometria feladattal: Meg kell adni annak a körnek(vagy köröknek) az egyenletét, amelyek áthaladnak (9;-2) ponton és érintik az x és y tengelyt is. Tudnátok segíteni, hogy hogy kell ezt kiszámolni?

[1232] HoA2009-05-27 13:38:52

1) Legyen a ka,kb,kc körök közös metszéspontja M, páronkénti második metszéspontjuk Mab,Mbc,Mca. Vegyük fel ka M-et nem tartalmazó MabMca ívén a Pa pontot. Legyen a PaMab egyenes és kb másik metszéspontja Pb , PbMbc egyenes és kc másik metszéspontja Pc . Könnyen belátható, hogy PcMca és McaPa párhuzamosak, vagyis egy egyenesen vannak. Különböző Pai -kat választva a keletkező PaiPbiPci háromszögek hasonlóak, hiszen például a Pai -knál fekvő szögek egyenlőek ka kör MabMca ívéhez tartozó kerületi szögével. Csak azt kell belátni, hogy ezen háromszögek csúcsai előállnak a feladatkitűzésben szereplő azonos, állandó szögsebességgel haladó futópontok egyidejű helyzeteként. Vegyünk fel két, 1-es és 2-es indexszel jelölt háromszöget. Pa1MabPa2 és Pb1MabPb2 szögek egyenlőek (csúcsszögek), a két körben a megfelelő ívhez tartozó kerületi szögek. Így a Pa1Pa2 és Pb1Pb2 ívekhez tartozó középponti szögek is egyenlőek, vagyis Pa és Pb ugyanakkora szögelfordulással jutnak ka-ban és kb-ben az 1-es helyzetből a 2-esbe, háromszögeink előállnak a feladatban megadott módon.

2) Nemcsak vándorló PaPbPc háromszögünk egyes helyzetei, hanem a részüket képező PaMPc háromszögek is hasonlók, hiszen egy-egy szögük az MMca húrhoz tartozó kerületi szög ka-ban ill. kc-ben. PaPbPc háromszög akkor lesz a legnagyobb területű, ha PaPc oldala a leghosszabb, vagyis ha PaMPc háromszög a legnagyobb. Ez pedig akkor következik be, amikor az M csúcshoz tartozó magasság a legnagyobb. Az utóbbi nem lehet nagyobb MMca -nál, és egyenlő is csak akkor, ha MMca merőleges PaPc-re. Legyen a csúcsoknak ez a helyzete PaM,PbM,PcM . A merőlegességből következik, hogy MPaM és MPcM átmérők – és ez belátható MPbM -re is.

Két megjegyzés:

a) Lássuk be, hogy háromszögünk akkor is megtartja alakját, mikor a ka-n futó pont az M-et tartalmazó MabMca íven halad.

b) PaM,PbM,PcM összetartozó ponthármas helyzetéből következik, hogy mindhárom körben 180o-ot fordulva mindhárom futópont M-be jut. Tehát , mint arra [1231]-ben utaltam, a körök metszéspontjában felvett futópont nem tilos, sőt a közös metszéspontban felvett 3 futópont éppen a megoldást adja.

152/3 feladatnak kitűzhetjük annak bizonyítását, hogy más megoldás nincs.

Előzmény: [1230] sakkmath, 2009-05-20 14:37:23
[1231] HoA2009-05-21 14:50:32

Kikötéseddel sokat segítettél. Mi van, ha valaki nem tartja be? ( Nem kell válaszolni. )

Előzmény: [1230] sakkmath, 2009-05-20 14:37:23
[1230] sakkmath2009-05-20 14:37:23

Az alábbiakban közlöm egy saját feladatomat. Nemrég rájöttem, hogy sajnos elődös: egy régi, közismert versenyfeladat következményének - vagy átfogalmazásának - is tekinthető.

152. feladat:

Egy adott ponton áthalad három, egymást páronként metsző kör. Mindegyik körön felveszünk egy-egy – a körök metszéspontjaitól különböző – pontot, melyek azonos körüljárási irányban befutják a körüket úgy, hogy egyszerre indulnak és azonos, állandó szögsebességgel haladnak.

1) Bizonyítsuk be, hogy e három pont felvehető úgy, hogy az általuk meghatározott háromszögek a teljes mozgás alatt hasonlóak legyenek egymáshoz.

2) Az így kapott háromszögek közül melyiknek a legnagyobb a területe?

[1229] Editkesss2009-05-17 15:10:56

De igen. A geometria nem az erősségem. (Ezeket a feladatokat már megoldottam . Abban viszont nem vagyok biztos, hogy jó is.) Ezért kértem segítséget!

[1228] Euler2009-05-17 13:28:10

A feladataid nagyon ismerősek, véletlenül nem euklideszi geometria gyakorlat SZTE-n?

Előzmény: [1227] Editkesss, 2009-05-17 09:10:21
[1227] Editkesss2009-05-17 09:10:21

Köszönöm szépen a segítséget! :) De lenne még egy-két feladatom! Az első: Tetszőleges e egyenes esetén jelölje: "Te" az e-re való tükrözést. Mutassuk meg, hogy ha "a" és "b" párhuzamos egyenesek, továbbá TaTbTcTaTbTc= identitás, akkor "c" is párhuzamos a-val és b-vel.

Második: Mutassuk meg, hogy ha az A1A2A3A4 és B1B2B3B4 négyszögek paralelogrammák, az AiBi szakasz Ai-hez legközelebbi negyedelőpontja Ni (i=1,2,3,4), akkor az N1N2N3N4 négyszög is (esetleg elfajuló) paralelogramma.

Előre is köszönöm!:)

[1226] Euler2009-05-16 22:21:09

A feladatot megpróbálom általánositva megoldani, vegyünk két nem egybeeső pontot, ekkor keressük azt a pontot a sikon, amelytől vett távolságaik négyzetösszege minimális, könnyen ellenőrizhető, hogy pont a két pontot összekötő szakasz felezőpontja lesz, pl. koordinátageometriával könnyen kijön,legyen A(a1;a2), B(b1;b2), a keresett pont: P(x;y), innen már csak egy másodfokú kifejezésnek kell vizsgálni a szélsőértkét, adódik az eredmény.Ez máshogy is kijöhet, bár itt nem biztos, hpogy "érdemes" igy gondolkodni, de ha mégis igy tesszük, akkor könnyen általánositható a probléma. Tudjuk ugyanis azt, hogy bármely háromszögben a szokásos jelölésekkel: 4sc2=2a2+2b2-c2(ez elég ismert összefüggésnek tekinthető,remélem.),a PAB háromszögre ezt felirva, kapjuk, hogy akkor lesz minimális a négyzetösszeg, ha a felezőponttól vett távolság minimális, máris adódik az eredmény. Kicsit továbblépve, vegyünk egy háromszöget(az egyszerűség kedvéért nem elfajulót), legyenek a csúcsai:A(a1;a2), B(b1;b2), C(c1;c2). keressük azt a pontot a sik egy adott egyenesén , amelytől vett távolságnégyzetösszeg minimális. Itt is hasonlóan eljárva, ráhúzodik a súlypontra az, ami előbb a felezőpontra húzodott rá,emiatt pedig az egyenesen lévő merőleges vetülete lesz a megfelelő pont.(Remélem érthető).Eljutottunk a Te problémádhoz, innen már "könnyű" elbánni vele, hiszen tekintsük az EBA és ECD háromszögeket, adódik, hogy akkor lesz mionimális a négyzetösszeg, ha az E-nek a CD és AB oldalak feletzőpontjától vett négyzetösszeg minimális, ez pedig használva az előzőeket, pontosan akkor lesz, ha a két felezőpont felezőpontjától vett távolság minimális, ez a pont pedig éppen a tetraéder súlypontja, ezt pl. vektorokkal lehet igazolni nagyon könnyen, igy itt a súlypontot kell merőlegesen vetiteni a sikra, ez lesz a keresett pont. Nyilván ha emeljük a dimenziószámot, hasonlóan adódik a feladat megoldása, csak maximum nem tudjuk elképzelni, hogy miről is szól a feladat. :-) remélem, tudtam segiteni a probléma megértésében.

Előzmény: [1225] Editkesss, 2009-05-16 18:57:42
[1225] Editkesss2009-05-16 18:57:42

Hello, nekem egy olyan feladatom lenne, hogy adva van egy S sík és egy abcd tetraéder. veszünk egy E pontot . kérdés: Az S sík mely E pontjára lesz ez AE*2+BE*2+CE*2+DE*2 a kifejezés értéke a legkisebb? *2 a négyzetet jelöli!

[1224] kandi2009-05-15 17:07:59

Nagyon szépen köszönöm ez így nagyon sokat segített! KÖSZÖNÖM

Előzmény: [1223] Vonka Vilmos Úr, 2009-05-15 15:46:44
[1223] Vonka Vilmos Úr2009-05-15 15:46:44

Akkor tényleg csak néhány ötlet:

1. és 2. Az adott ponton keresztül, amelynek a képét keressük, tekintsünk egy tetszőleges segédegyenest, és próbáld először ennek a képét megszerkeszteni.

3. A megfelelő egyenespár segítségével először határozzuk meg a kollineáció tengelyét, majd használjuk ki azt a tényt, hogy a centrum és az eltűnési egyenes távolsága megegyezik a tengely és az ideális egyenes képének távolságával.

4. A megfelelő egyenespár és egy pont ismeretében ismét először keressük meg a tengelyt. Majd válasszunk egy tetszőleges segédpontot azon az egyenesen, amelyiknek adott a képe; és próbáld a választott pont képét megszerkeszteni.

5. Az ABCD négyszög képe akkor lesz paralelogramma, ha a szemköztes oldalegyenesek metszéspontjaihoz a kollineáció ideális pontot rendel. Ez alapján határozd meg először az eltűnési egyenest. A tengely meghatározásához pedig tekintsünk ismét egy segédegyenest azon a ponton keresztül, amelynek a képe adott, és szerkesszük meg először a segédegyenes képét.

Remélem, innentől már menni fog!

Előzmény: [1218] kandi, 2009-05-13 07:55:44
[1222] kandi2009-05-15 10:42:38

Köszönöm a linket, bár nem igazán értem az anyagot:( Amúgy centrális kollineációval oldhatók meg elvileg a feladatok és mi a projektív geometriával foglalkozunk. Ha mégis lenne még egy kis segítség mert én már kifuladtam a sz ötletekből?!

[1221] jonas2009-05-14 21:55:55

Én inkább ábrázoló geometriára tippelnék, bár szerencsére nekem nem kellett ilyesmit tanulnom, úgyhogy nem vagyok biztos, hogy valóban erről van szó.

Előzmény: [1219] HoA, 2009-05-14 15:22:08
[1220] HoA2009-05-14 16:14:44

Gratulálok! Igen, erre a megoldástípusra gondoltam! A D ponthoz tartozó bizonyításra egy másik változat: A definíció szerint D a P9P18 és P6P17 átlók metszéspontja, és azt kell igazolni, hogy P4P16-on is rajta van. DBO\angle=20o ,mint a P6P8 ívhez tartozó kerületi szög. BOD\angle=20o ,mint a P17P18 ívhez tartozó középponti szög, OBD\Delta egyenlőszárú. P14OP17\angle=60o ( 3 ívegység középponti szöge ) , P14OP17\Delta szabályos. Így BDOP14 deltoid, P17P14D\angle=30o . Így D rajta van a P14P17-tel 30o-ot bezáró P2P14 átlón és ennek P9P18 -ra vett tükörképén, P4P16-on is.

Előzmény: [1216] sakkmath, 2009-05-12 15:24:47
[1219] HoA2009-05-14 15:22:08

Szia Kandi! Ez itt a KöMaL Geometria fóruma. Feladataid úgy látom, nem egészen a középiskolai geometria témájába tartoznak, ami még önmagában nem baj. Csak az nem világos, miről is van szó. A használt fogalmak alapján úgy gondolom, valamilyen projektív geometriai kurzusra jársz. A feladatok esetleg mind centrális kollineáció témába esnek? Ha nem ismered, talán segít ez a jegyzet:

http://www.jgytf.u-szeged.hu/~krisztin/projektiv.doc

Előzmény: [1218] kandi, 2009-05-13 07:55:44
[1218] kandi2009-05-13 07:55:44

Szia Mindenki! Én még itt új vagyok, úgyhogy nem nagyon tudom, hogy hogyan működik, de nagy segítségre lenne szükségem. Van 5 db szerkesztési példám. Ami elvileg nagyon könnyű, de én nem tudtam megcsinálni, ha valaki segítene megköszönném. Leírom a feladatokat: 1., adott : tengely, centrum, eltűnési egyenes.Szerk. meg tetszőleges pont képét! 2., adott : tengely, centrum, ideális egyenes képe.Szerk. meg tetszőleges pont képét! 3., adott : az eltűnési egyenes, az ideális egyenes képe, és egy egymásnak megfelelő egyenespár.Szerk. meg a centrumot és a tengelyt! 4., adott : egy megfelelő pontpár, egy megfelelő egyenespár és a tengely egy pontja. Szerk. meg a centrumot és a tengelyt! 5., adott : a centrum és egy megfelelő pontpár. Határozzuk meg a tengelyt úgy, hogy egy előre adott ABCD négyszög képe paralelogramma legyen! Előre is köszönöm:)

[1217] MTM2009-05-12 18:06:51

151.feladat:

ABC háromszög körülírt körét, AB és AC oldalt rendre D, E, G-ben érinti egy kör. Bizonyítsuk be, hogy BDC szög felezője felezi EG szakaszt.

[1216] sakkmath2009-05-12 15:24:47

Rajz a megoldás 2. részéhez:

Előzmény: [1215] sakkmath, 2009-05-12 15:23:45
[1215] sakkmath2009-05-12 15:23:45

A megoldás 2. része:

Előzmény: [1214] sakkmath, 2009-05-12 15:22:41
[1214] sakkmath2009-05-12 15:22:41

Rajz a megoldás 1. részéhez:

Előzmény: [1213] sakkmath, 2009-05-12 15:20:18
[1213] sakkmath2009-05-12 15:20:18

A HoA [1208]-as hozzászólásában kitűzött 150. feladat két feladatnak tekinthető. Elnézést, de sajnos csak négy részletben tudom feltenni a megoldást: a képek 50 KB-os határával a megfelelő képminőség ugyanis nem összeegyeztethető.

A megoldás 1. része:

Előzmény: [1208] HoA, 2009-04-29 15:00:17
[1212] sakkmath2009-05-12 09:40:01

Tetszik a feladat, s már kész vagyok egy teljes, elemi geometriai bizonyítással. Az utolsó simításoknál tartok, legkésőbb holnap reggel felteszem.

Előzmény: [1208] HoA, 2009-04-29 15:00:17
[1211] BohnerGéza2009-05-11 23:22:32

Segítség a 150. feladat egy lehetséges megoldásához: Az E és D pontot máshogy definiálva lássuk be addíciós tételek következménye segítségével, hogy megfelelnek az eredeti definíciónak.

Egyelőre megpróbálok más jellegű (számomra szebb) megoldást is kitalálni.

Előzmény: [1208] HoA, 2009-04-29 15:00:17
[1210] HoA2009-05-07 22:43:48

Mit néztél el? Szerintem a feladat jó. Nyilván sokunknak beugrik a hetedik, azonos sugarú kört és egymást érintő hat kör, és ezek befoglaló köre. De azt nem sikerült bizonyítanom, hogy a hetedik kör elhagyásával nem adható jobb megoldás - kisebb befoglaló kör.

Előzmény: [1209] BohnerGéza, 2009-05-07 00:35:55
[1209] BohnerGéza2009-05-07 00:35:55

A 148. feladat megoldása: A keresett sugár 12 cm. Ha valaki tud jobb feladatot, tűzze ki! (Elnéztem.)

Előzmény: [1203] BohnerGéza, 2009-04-08 05:14:08
[1208] HoA2009-04-29 15:00:17

Úgy látom nagy a csend. Hadd tűzzem ki 150. feladatnak a KÖMAL egyik régi számában talált problémát –a megoldások után pontosan megjelölöm a forrást.

Az ABC egyenlőszárú háromszögben AC = BC és a C-nél lévő szög 80o . A háromszög belsejében kijelöljük azt a D és E pontot, amelyre DAB\angle=EAB\angle=10o,DBA\angle=30o,EBA\angle=20o . Bizonyítsuk be, hogy DCB\angle=10o és ECB\angle=20o.

A téma olvasói könnyen rájönnek, miért tetszett meg ez a feladat.

[1207] BohnerGéza2009-04-17 13:56:24

A 149. feladathoz: 2. rész.

Az előző [1206] hozzászólás folytatása.

A megoldásban nem törekedtem a teljességre, a legegyszerűbb megfogalmazásra, inkább a gondolatmenet bemutatására.

Jó lenne, ha valaki szebben megfogalmazott, vagy más jellegű megoldást is adna!

Vegyük észre a kapcsolatot az itt és HoA[1204] hozzászólásában látottak közt!

Előzmény: [1206] BohnerGéza, 2009-04-17 13:43:59
[1206] BohnerGéza2009-04-17 13:43:59

A 149. feladathoz: 1. rész.

Folytatása a következő hozzászólásban.

[1205] BohnerGéza2009-04-17 13:29:51

Nekem is hiányérzetem volt janomo[1200] hozzászólásával kapcsolatban. Igaz így legalább HoA pontos, érthető megfogalmazásában láthattuk a megoldást!

Példát mutatok egy kevésbé szépre a következő hozzászólásban, az eredeti feladat kapcsán. Remélem, látható lesz belőle, hogy a szép megoldások is esetleg hosszabb gondolkodás után, sok munka árán, nem véletlenszerűen jönnek össze. Az így elért eredmény, sőt részeredmény is örömet adhat. (Ráadásul nem kell közben pl. négykerekű motoros marhasággal -nem jut eszembe a neve - tönkretenni a környezetet.)

149. feladat: A Surányi János emlékverseny 2. feladata: Az ABC háromszög beírt köre az AB és AC oldalakat rendre a D és E pontban érinti. A beírt körnek és az AEB háromszög köré írt körnek E-től különböző közös pontja legyen F, a D pont merőleges vetülete az EB egyenesen G. Igazoljuk, hogy 2ABEszög=BFGszög.

(Elnézést, a továbbiakban ezeket az eredeti jelöléseket használom!)

Előzmény: [1204] HoA, 2009-04-17 09:10:46
[1204] HoA2009-04-17 09:10:46

Szerintem nem érdemes annyival elintézni az inverziós megoldást, hogy „inverzió a B pontra és kész”. Más feladatoknál is felhasználható például, hogy három, a póluson nem áthaladó egyenesen fekvő pont képe egy, a póluson átmenő körön van, és viszont: ha négy pont húrnégyszöget alkot – egy körön van – akkor egyiküket pólusnak választva a másik három pont inverz képe egy egyenesen fekszik.

Legyen tehát az inverzió pólusa B, alapköre a k-t merőlegesen metsző kör. Ekkor k képe önmaga, A és A’ egybeesik, F képe az AB egyenes F’ pontja, melyre F’A = AB, AB Thálesz-körének képe az AB-re merőleges e egyenes, D képe, D’ BD és e metszéspontja, I képe a BI egyenes k-val alkotott második metszéspontja. CIF egyenes képe a B’, F’, I’ pontok által meghatározott kör. Ennek F’B húrja, e húrfelező merőlegese, tehát szimmetriatengelye. e k-nak is szimmetriatengelye, így a két kör metszéspontjai, C’ és I’ egymás tükörképei e-re. Ez igaz az F’, B pontpárra is, tehát a BD’I’ egyenes e-re vett tükörképe az F’D’C’ egyenes. A fentiek szerint ezért BCDF húrnégyszög, körülírt körében az egyenlő BF és DF húrokhoz egyenlő kerületi szögek tartoznak: DCF\angle=FCB\angle.

Előzmény: [1200] janomo, 2009-04-04 12:18:33
[1203] BohnerGéza2009-04-08 05:14:08

148. feladat: Mekkora annak a legkisebb körnek a sugara, amelyben átfedés nélkül elfér 6 db 4 cm sugarú kör? (Érintkezés lehet.)

[1202] BohnerGéza2009-04-08 05:10:40

Érdemes HoA [1198]-ban lévő megjegyzése alapján is végiggondolni a megoldást!

Előzmény: [1201] HoA, 2009-04-07 22:58:30
[1201] HoA2009-04-07 22:58:30

Tényleg kár, hogy másokat nem érdekelnek ezek a jó kis feladatok. A 147. feladathoz: D rajta van AB Thálesz körén, AF = DF = BF. A szelőtételből FD2=FA2=FI.FC . FID és FDC \Delta-ek hasonlók, FBD \Delta egyenlőszárú, DCF\angle=FDB\angle=DBF\angle, BCDF húrnégyszög, körülírt körében az egyenlő BF és DF húrokhoz egyenlő kerületi szögek tartoznak: DCF\angle=FCB\angle

Előzmény: [1197] BohnerGéza, 2009-04-01 19:15:28
[1200] janomo2009-04-04 12:18:33

Inverzió a B pontra és kész.

Előzmény: [1197] BohnerGéza, 2009-04-01 19:15:28
[1199] BohnerGéza2009-04-03 11:43:26

Írtam, hogy belső szögfelező?!

Előzmény: [1198] HoA, 2009-04-03 11:38:44
[1198] HoA2009-04-03 11:38:44

A mellékelt ábra nem csak gonoszkodás, talán a megoldáshoz is segítséget nyújt :-)

Előzmény: [1197] BohnerGéza, 2009-04-01 19:15:28
[1197] BohnerGéza2009-04-01 19:15:28

Köszönöm HoA! Elírtam. Helyesen a feladat:

A Surányi János emlékverseny 2. feladata alapján.

147. feladat: Érintse a k kör az AB egyenest az A pontban és legyen C a k egy A-tól különböző pontja, F az AB szakasz felezőpontja. Az FC messe még k-t az I pontban, az A-nak a BI-re eső merőleges vetülete D.

Bizonyítandó, hogy CF felezi a DCB szöget.

Előzmény: [1196] HoA, 2009-04-01 16:15:55
[1196] HoA2009-04-01 16:15:55

Az ábra alapján inkább a cáfolat mint a bizonyítás látszik esélyesebbnek.

Előzmény: [1194] BohnerGéza, 2009-03-31 20:48:12
[1195] BohnerGéza2009-03-31 20:57:04

Örülnék, ha nem csak HoA kapcsolódna be a 144. és 144.b feladat megoldásába.

Hogy jellemezhető pl. HoA 6 megoldása, hány és milyen megoldást adó P lehet még a háromszögön kívül?

Előzmény: [1193] HoA, 2009-03-28 14:32:44
[1194] BohnerGéza2009-03-31 20:48:12

A Surányi János emlékverseny 2. feladata alapján.

147. feladat: Érintse a k kör az AB egyenest az A pontban és legyen C a k egy A-tól különböző pontja, F az AB szakasz felezőpontja. Az FC messe még k-t az I pontban, az A-nak a BI-re eső merőleges vetülete D.

Bizonyítandó, hogy CF felezi a DFB szöget.

[1193] HoA2009-03-28 14:32:44

144.b feladathoz: Az ABC háromszögön belüli P pontokra a talpponti DEF háromszögben a megfelelő pontok hat különböző helyzetben lehetnek szabálytalan hegyesszögű háromszög esetén, lásd [1190] ábráját. A [1189]-ben vázolt szerkesztést elvégezve mind a hat pontra kapunk megoldást az ábra szerint.

Előzmény: [1192] BohnerGéza, 2009-03-25 04:35:44
[1192] BohnerGéza2009-03-25 04:35:44

a 144. feladattal kapcsolatban:

HoA nagy ötlete után példát mutatok az adott ABC háromszöghöz olyan P pont szerkesztésére, amelyhez adott QRS háromszöghöz hasonló talpháromszög tartozik:

Az ábra Q’PS’ szöge 180 fok-alfa, egyenlő a QP’S szöggel, így P’ számára adott vonal a QS szakasz 180 fok-alfa szögű látóköre. Hasonlóan pl. az RQ 180 fok-béta látóköre is, így P’ szerkeszthető.

A Pa pont SP’ távolságra van az AC-től és QP’-re az AB-től. P-nek az APa-n (és hasonlóan a BPb-n) kell lenni. …

Természetesen több megoldása lehet (van) a feladatnak. 144.b feladat: Adjuk meg a lehetséges megoldások számát!

Előzmény: [1188] HoA, 2009-03-22 19:52:41
[1191] Maga Péter2009-03-22 22:33:02

Szia! - 2 hónap késéssel, de...

Látom, hogy már kaptál segítséget, de ha dobsz egy e-mailt, akkor tudok küldeni egy anyagot. Nem egészen elemi, de annak idején (tavaly vagy tavalyelőtt) a debreceni Fazekas Gimnázium matek önképzőkörén mondtam el, és értették, legalábbis úgy tettek:).

Előzmény: [1166] edu, 2009-01-23 09:22:12
[1190] HoA2009-03-22 20:09:30

Az ABC belsejében fekvő P pontokhoz olyan DEF háromszög tartozik, melynek egy-egy oldala P-ből \pi-\alpha ill. \pi-\beta szög alatt látszik. A 3. ábrán ezért a DEF háromszög oldalainak \pi-\alpha (zöld) ill. \pi-\beta (kék) látószögű köríveit és azok metszéspontjait vizsgáljuk. A szimmetria kedvéért feltüntettük a \pi-\gamma (lila) látószögű köríveket is, ezek természetesen áthaladnak ugyanazokon a metszéspontokon. A hat metszéspont közül az egyik DEF magasságpontja. Tudjuk, ez felel meg annak, ha P gyanánt ABC körülírt körének középpontját választjuk. Két másik metszéspont DEF első ill. második Brocard pontja (Könnyen belátható, hogy a Brocard pontokból két oldal látószöge \pi-\alpha ill. \pi-\beta) . Kérdések: DEF Brocard pontjait választva a szerkesztés eredményeként kapott P ABC-nek is Brocard pontja lesz? Kapunk-e megfelelő P pontot a 3. ábra fennmaradó három látószög-körív metszéspontjából?

Előzmény: [1186] BohnerGéza, 2009-03-19 19:55:52
[1189] HoA2009-03-22 20:04:53

A második ábrán a H2 DEF háromszög oldalainak a háromszög belseje felé eső \alpha (zöld) , \beta (kék) ill. \gamma (lila) látószögű köríveit és azok metszéspontjait vizsgáljuk. Az R pont jó lenne az 1. ábra P pontjának, csak \beta és \gamma sorrendje fordított. Ezért szerkesztésünk alapja a H2 háromszög H3 tükörképe. Legyen EDR szög = \delta. Vegyük fel az 1. ábrán az AC szakasz C-n túli meghosszabbításán a D’ pontot. A D’-pontban AC-re emelt merőlegessel annak A-t tartalmazó oldalán \delta szöget bezáró egyenes és AB metszéspontja E’ . A H3 –hoz hasonló D’E’F’ H4 háromszöget A-ból vetítsük úgy, hogy F’ F képe BC-re essen. Ekkor a H4 ( = H2 ) DEF háromszög megfelel, P az AC-re D-ben és az AB-re E-ben emelt merőlegesek metszéspontja. Kérdések: Kapunk-e így megoldást a szemközti, A csúcsú végtelen síktartományban? A 2. ábra (\alpha, \beta) látószögű Q pontja ad-e megoldást az 1. ábra AB szakaszának C-t nem tartalmazó oldalán lévő síktartományban ? Hát a C csúcsú végtelen síktartományban? A 2. ábrán DEF piros körülírt körének D-t nem tartalmazó EF ívén minden pontból a DEF háromszög egy-egy oldala \beta ill. \gamma szög alatt látszik. Megfelenek-e ezek a pontok R szerepére?

Előzmény: [1186] BohnerGéza, 2009-03-19 19:55:52
[1188] HoA2009-03-22 19:52:41

Nevezzük a megfelelő P pont vetületei által meghatározott háromszöget P talpponti háromszögének. Mit állapíthatunk meg az adott ABC háromszög oldalegyenesei által hét tartományra osztott sík egyes részeiben a P-ből a talpponti háromszög csúcsaiba vezető szakaszok által bezárt szögekről? Az 1. ábráról leolvasható, hogy 2-2 tartományban ezek a szakaszok egymással nem egymás szögtartományába eső , szokásos szögmérési irányban mért ( \alpha,\gamma) , ( \gamma,\beta ) illetve ( \beta,\alpha ) szögeket zárnak be. A háromszög belsejében a megfelelő szakaszok szöge pl. ( \pi\alpha,\pi\beta )

A keresendő P pontok lehetséges helyzetét vizsgáljuk nem az ABC, hanem a talpponti, DEF háromszöghöz képest. Ezzel a keresett P pontok megszerkesztésének egy lehetséges útját is kijelöljük. Általános hegyesszögű ( 45, 60 75 fokos ) háromszöget választottam. ( Majd meg kell vizsgálni, hogy módosulnak az eredmények egyenlőszárú, egyenlőoldalú, derékszögű, tompaszögű, vagy akár más általános hegyesszögű háromszög esetében. ) Ha van megfelelő P pont az 1. ábra BC szakaszának A-t nem tartalmazó oldalán lévő síktartományban, akkor P-ből a DEF háromszög egy-egy oldala \beta ill. \gamma szög alatt látszik, mégpedig úgy, hogy P DEF egy-egy oldalának \beta ill. \gamma látószögű, a háromszög belseje felé eső körívén van. ( Mikor jöhetnek szóba a külső körívek? )

Előzmény: [1186] BohnerGéza, 2009-03-19 19:55:52
[1187] fityfiritty2009-03-21 20:17:29

Köszönöm.

Itt egy új, a számozottak között 146. feladat:

A hegyesszögű APD\Delta AP, illetve PD oldalának tetszőleges pontja rendre B, illetve C. Az ABCD négyszög átlói a Q pontban metszik egymást. M1, illetve M2 rendre az APD\Delta, illetve a BPC\Delta magasságpontja. Az ABQ és CDQ háromszögek körülírt köreinek Q-tól különböző metszéspontja legyen X, a BCQ és ADQ háromszögekre ugyanígy kapott pont pedig legyen Y. Bizonyítsuk be, hogy: ha az M1, M2 és X pontok egy e egyenesre esnek, akkor Y \ine.

Előzmény: [1182] HoA, 2009-03-16 16:59:04
[1186] BohnerGéza2009-03-19 19:55:52

A 144. feladathoz: Az ábrán látható két olyan (a két kék ponthoz tartozó) az eredetihez hasonló "talpháromszög", melyekben az AB-nek megfelelő oldal az AB és az AC egyenest köti össze.

Az előző hozzászólás ábráján is két ilyen van, a zöld és a barna ponthoz tartozó. ...?

Előzmény: [1185] BohnerGéza, 2009-03-19 17:53:44
[1185] BohnerGéza2009-03-19 17:53:44

A 144. feladathoz: Az ábrán látható három, az eredetihez hasonló "talpháromszög".

A barna és a kék pont a háromszög Brocard-pontja. Azt nem nagyon nehéz bizonyítani, hogy a hozzájuk tartozó talpháromszögek megfelelnek a feladatnak, hogy ez a kettő egybevágó azt nehezebb.

A körülírt kör kp-jához tartozó középvonali háromszög is jó.

Az ábra zöld pontja is jó. ...?

Előzmény: [1173] BohnerGéza, 2009-02-16 20:08:53
[1184] HoA2009-03-18 12:06:30

Lenne egy kis gyakorlati problémám, úgy vélem ehhez a témához tartozik:

Adottak egy "majdnem derékszögű" háromszög a, b, c oldalai. Keressük meg a legjobban közelítő derékszögű háromszöget abban az értelemben, hogy

(a+\Deltaa)2+(b+\Deltab)2=(c+\Deltac)2

teljesüljön, úgy, hogy \Deltaa2+\Deltab2+\Deltac2 minimális legyen.

Előzmény: [1183] SmallPotato, 2009-03-17 22:57:39
[1183] SmallPotato2009-03-17 22:57:39

Találtam egy olyan irodalmat, amelyik talán érthetően vezeti le a dolgot. A 60-61. oldalakon van a Téged érintő/érdeklő rész. Ott y,x1,x2 változókról beszél, ezek a Te példádban (sorrendben) z,x,y.

A mintafeladatodat megcsináltam ezeknek az egyenleteknek a segítségével (a 61. oldal tetején a három egyenlet - mint egyenletrendszer - megoldása), és Excelben is (LIN.ILL függvény). Az eredmények teljesen megegyeznek.

A regressziós sík egyenlete a kis példádra, 6 értékes jegyre:

z=0,744949x+27,6667y-5898,09

Írj mailt, megküldöm az Excel-táblát.

Előzmény: [1181] david20, 2009-03-16 12:01:19
[1182] HoA2009-03-16 16:59:04

Microsoft Visio. Sajnos nem tudok ingyenes változatról :-(

Előzmény: [1178] fityfiritty, 2009-03-11 12:17:28
[1181] david202009-03-16 12:01:19

Üdvözöllek!

Elolvastam a linkelt doksikat, és sok példát is találtam a neten, de valahogy nem akar kijönni...

Tudna valaki egy kis példán keresztül segíteni a "regressziós sík" kiszámításában az alábbi pontokra.

Előre is köszönöm.

Előzmény: [1177] SmallPotato, 2009-03-11 01:09:43
[1180] lorantfy2009-03-15 10:47:04
Előzmény: [1154] farkasb, 2008-12-05 16:47:37
[1179] Gábor19052009-03-11 22:54:12

Üdv. A következő kérdésre szeretnék választ kapni: Van-e olyan konvex 5, vagy attól többszög, aminek több mint 4 belső szöge 120°, vagy attól kisebb. Minden ötlet nagyon érdekel! Előre is köszönöm.

[1178] fityfiritty2009-03-11 12:17:28

Helló, HoA!

A h.f.-ekhez szívesen használnék ilyen progit, amivel ez a rajz készült. Mi a neve? Köszi, előre is.

Előzmény: [1170] HoA, 2009-02-05 20:03:36
[1177] SmallPotato2009-03-11 01:09:43

Hogy a pontok mennyire pontosan vannak egy körvonalon, az - szerintem - gyakorlatilag mindegy. Ami nem mindegy: hogy egy síkban vannak-e. Ha nincsenek, akkor regressziós síkot kell keresni. (Ellenkező esetben nyilván bármelyik 3 pont kifeszíti a keresett síkot.) Regressziós sík megtalálására az irodalomban (pl. itt) találsz több megközelítést is (kiemelés tőlem):

"... egy, a megadott pontokhoz leginkább simuló általános kiegyenlítő sík egyenletét határozza meg, úgy, hogy a pontok és a sík közötti z irányú távolságok négyzetösszegét minimalizálja ..."

vagy

"... a megadott pontokhoz legjobban simuló kiegyenlítő sík egyenletét számítja ki úgy, hogy a pontok és a sík közötti merőleges távolságok négyzetösszegét minimalizálja ..."

Amit még találtam hirtelen: A regressziószámítás gyakorlati kérdései

Ha a regressziós egyenes megkeresésének módját ismered (így van vajon?), nem sok gondod lesz a síkkal sem.

Előzmény: [1176] david20, 2009-03-10 13:12:11
[1176] david202009-03-10 13:12:11

Üdvözlök Mindenkit!

Egy kis segítségre lenne szükségem.

Adott egy térbeli koordináta rendszer, és 600 pont (X,Y,X), amik megközelítőleg (nem pontosan) egy kör körvonalán helyezkednek el. A kör síkja nem párhuzamos az XY síkkal.

A pontok a koordináta rendszerben való ábrázolása a képen látható.

Hogyan lehetne ezekre a pontokra egy síkot illeszteni?

Minden segítséget nagyon szépen köszönök.

David20

david20x@gmail.com

[1175] BohnerGéza2009-03-07 22:11:08

145. feladat:

[1174] HoA2009-02-25 17:27:42

Legyen a P pont ABC \Delta -re vonatkozó Simson egyenese s. Mivel a Feuerbach kört M-ből kétszeresére nagyítva a k körülírt kört kapjuk, a feladat második, A1B1C1 háromszöge ABC középponti háromszögének M-ből vett kétszeres nagyítása, vagyis ABC 180o-os elforgatottja a körülírt kör O középpontja körül. Forgassuk el az ABC \Delta-et és P-t 180o-kal O körül. ABC \Delta A1B1C1\Delta-be, P pedig k átellenes P1 pontjába kerül, s képe a vele párhuzamos s1 egyenes lesz, s1 így a P1 pont A1B1C1\Delta-re vonatkozó Simson egyenese . Ha most P1-et O körül 180o-kal elforgatva P-be visszük, A1B1C1\Delta-re vonatkozó Simson egyenese [1170] (1) szerint 90o-kal fordul el, új helyzete s2 merőleges lesz s1-re és így s-re is.

Előzmény: [1172] BohnerGéza, 2009-02-05 23:48:17
[1173] BohnerGéza2009-02-16 20:08:53

144. feladat: Adott háromszög esetén határozzuk meg azon pontok halmazát, melyeknek a három oldalegyenesre eső merőleges vetülete az eredetihez hasonló háromszöget határoz meg.

[1172] BohnerGéza2009-02-05 23:48:17

HoA szép megoldása - az (1) tétel ismerete - után már könnyebb a következő feladat!

143/2. feladat: Bizonyítandó, hogy egy nem derékszögű háromszögnek és a magasságpontja oldalfelezőpontokra vonatkozó tükörképei által kapott háromszögnek ugyanazon ponthoz tartozó Simson-egyenese merőleges egymásra!

Előzmény: [1170] HoA, 2009-02-05 20:03:36
[1171] HoA2009-02-05 20:07:30

Helyesbítek: GH az A pont DEF háromszögre vonatkozó Simson egyenese.

Előzmény: [1170] HoA, 2009-02-05 20:03:36
[1170] HoA2009-02-05 20:03:36

Azért nem várok egy hetet, mert az alábbi megoldás nem csak a középiskolai anyagra támaszkodik, kihasználja a Simson egyenesnek azt a tulajdonságát, hogy

a körülírt kör két , P és Q pontjához tartozó Simson egyenes szöge megegyezik a körülírt kör PQ ívéhez tartozó középponti szög felével. (1)

Ennek alapján elég az állítást a körülírt kör tetszőleges P pontjára belátni, hiszen P-t bármely más pontba mozgatva a két vizsgált háromszögre vonatkozó Simson egyenes ugyanazzal a szöggel fordul el, tehát megtartják merőlegességüket. Legyen az ABC háromszög magasságpontja M, az AM, BM, CM magasságoknak a körülírt körrel alkotott másik metszéspontjuk rendre D,E,F. ACF és EBA szögek egyenlőek, mert egy \alpha szögű derékszögű háromszög harmadik szögei. Ezért az AE és AF ívek egyenlőek, és a hozzájuk tartozó EDA és ADF kerületi szögek is. Legyen P = A. Ekkor az ABC háromszögre vonatkozó Simson egyenes az AD magasságvonal. A-ból ED-re és FD-re bocsátott merőleges talppontja G és H. ADG és ADH egybevágó derékszögű háromszögek, G és H egymás tükörképei AD-re, ezért GH, az A pont EFG háromszögre vonatkozó Simson egyenese merőleges AD-re.

Talán tud valaki közvetlen, az (1) tételt nem felhasználó megoldást.

Előzmény: [1169] BohnerGéza, 2009-02-04 14:46:10
[1169] BohnerGéza2009-02-04 14:46:10

143. feladat: Bizonyítandó, hogy egy nem derékszögű háromszögnek és a magasságpontja oldalakra vonatkozó tükörképei által kapott háromszögnek ugyanazon ponthoz tartozó Simson-egyenese merőleges egymásra!

[1168] edu2009-01-23 17:10:12

Köszönöm a segítséget. A szerkesztés már ok.; a bizonyítással még bírkózom.

Előzmény: [1167] SmallPotato, 2009-01-23 09:58:52
[1167] SmallPotato2009-01-23 09:58:52

A Mascheroni-féle szerkesztésekről (tételesen az általad kérdezettről is) olvashatsz pl. itt:

PowerPoint bemutató

vagy itt:

Adobe Readerrel olvasható PDF fájl

Előzmény: [1166] edu, 2009-01-23 09:22:12
[1166] edu2009-01-23 09:22:12

Hi! A csak körzővel való szerkesztésekről keresek irodalmat. Különös tekintettel érdekelne egy adott kör középpontjának megszerkesztése (csak körzővel),ill. ennek bizonyítása (ha lehet elemi úton).A segítséget előre is köszönöm.

[1165] BohnerGéza2009-01-21 09:49:55

Egy nedijan-éhoz hasonló feladat:

Szerkesztendő ABC háromszög, ha adott c, a-b és alfa. (Sokkal könnyebb, ha béta adott?)

Előzmény: [1161] nedijan, 2009-01-11 10:47:13
[1164] HoA2009-01-14 17:47:33

Vagy ugyanez másként: CD szakasz D végpontjában vegyük fel a CD-hez alfa/2 szögben hajló e egyenest. Ezt C középpontú a sugarú körívvel metszve kapjuk B-t. A diszkusszió így talán egyszerűbb: A megadott adatok között milyen összefüggés áll fenn, ha 0, 1 ill. 2 megoldást kapunk?

Előzmény: [1162] jenei.attila, 2009-01-11 13:38:36
[1163] nedijan2009-01-11 15:50:31

Köszönöm a segítséget. Jani

Előzmény: [1162] jenei.attila, 2009-01-11 13:38:36
[1162] jenei.attila2009-01-11 13:38:36

b-t hosszabítsd meg A csúcson túl c-vel. Az így kapott végpontot jelöld D-vel, vagyis a CD szakasz hossza b+c lesz. Tekintsd a DBA egyenlő szárú háromszöget, amelynek A csúcsba futó szárai c hosszúságúak, a DB alapon fekvő szögei pedig alfa/2 nagyságúak, mivel az A-nál lévő külső szög alfa nagyságú, ami az ABC keresett háromszögünk A-nál lévő adott szöge. A szerkesztés menete: megszerkeszted az a, b+c oldalakkal bíró, alfa/2 nagyságú a-val szemközti szögű háromszöget (a fölé alfa/2 látószögkörív, amit a egyik végpontjából b+c-vel elmetszel; de gondolom ezt nem kell magyarázni). Megkapod, a már említett D pontot. felveszed B-ben a DBA alfa/2 nagyságú szöget. Ahol a szögszár elmetszi a CD szakszat, az lesz az A csúcs. Remélem érthető volt.

Előzmény: [1161] nedijan, 2009-01-11 10:47:13
[1161] nedijan2009-01-11 10:47:13

Sziasztok!

Lenne egy egyszerű geometriai szerkeztésem, de sehogy nem jövök rá a megoldásra. Adott b+c, a és az a-val szemközti szög. Megszerkezthető-e a háromszög, ha igen adja meg a szerkesztés menetét! Ebben kérem a segítségeteket. Előre is köszönöm.

Jani

[1160] farkasb2008-12-07 18:56:52

Köszönöm a válaszokat!

[1159] Fálesz Mihály2008-12-06 22:08:11

Az AB irányű egységvektor \frac{\vec{AB}}{|AB|}. Az AP szakasz vetületének, azaz AX-nek az előjeles hossza az AB egyenesen \vec{AP}\cdot\frac{\vec{AB}}{|AB|}. Tehát,


\vec{AX} =
\left(\vec{AP}\cdot\frac{\vec{AB}}{|AB|}\right)\cdot
\frac{\vec{AB}}{|AB|} =
\frac{\vec{AP}\cdot\vec{AB}}{|AB|^2}\cdot\vec{AB}.

Előzmény: [1157] HoA, 2008-12-06 19:00:36
[1158] HoA2008-12-06 20:33:36

A paraméteres alakot talán úgy a legegyszerűbb felhasználni, hogy AQ és PQ merőlegességét írjuk fel. t-vel paraméterezve AQ = Q - A = ( 4t ; -2t; -3t ) PQ = Q - P = Q - A + A - P = ( Q - A ) - u = ( 4t -6 ; -2t + 4; -3t + 1) . A kettő szorzata 0. 4t ( 4t - 6 ) + 2t ( 2t - 4 ) + 3t ( 3t - 1 ) = 16t2-24t+4t2-8t+9t2-3t=29t2-35t=0 . t-vel oszthatunk - a 0 hosszúságú AA vektor bármire merőleges - , t = 35 / 29 , ami az előző levezetés számszerű eredményét adja.

Előzmény: [1156] BohnerGéza, 2008-12-06 17:47:56
[1157] HoA2008-12-06 19:00:36

Ne ijesszük el farkasb-t, nem kell ide szélsőérték-számítás. Az X pont az AB szakaszt az AP szakasz és a PB szakasz AB-re eső vetületeinek arányában osztja. Legyen \vec{AB} = {\bf v},  \vec{AP} = {\bf u}. Ekkor \vec{PB} = {\bf v - u} . X = A + m ( B - A ) = m B + ( 1 - m ) A , ahol m és ( 1 - m) aránya az u és a v - u vektor v -re vonatkozó vetületeinek aránya. Innen

X = \frac{\bf uv}{\bf vv}  B + \frac{\bf (v-u) v}{\bf vv} A

Számadatainkkal u = ( 6;-4;-1 ), v = ( 4;-2;-3 ), v-u = ( -2; 2; -2), uv = 35, (v-u)v = -6, v v = 29.

X = \frac{35}{29} B - \frac{6}{29} A = (6,827586207;6,586206897;0,379310345) . vegyük észre, hogy az együtthatók összege 1, tehát X valóban az AB egyenes pontja és m > 1, így az ábrával ellentétben X az AB szakasz B-n túli meghosszabbítására esik.

Előzmény: [1154] farkasb, 2008-12-05 16:47:37
[1156] BohnerGéza2008-12-06 17:47:56
Előzmény: [1155] Valezius, 2008-12-06 10:55:08
[1155] Valezius2008-12-06 10:55:08

Az egyenes pontjai ilyen alakúak: (2+4x, 9-2x, 4-3x) P távolsága egy adott ponttól: (8,5,3)-előbbi. Azaz (6+4x, -4-2x, -1-3x) Azt az x-et keressük amire a vektor hossza minimális. A hossza nem más, mint a koordináták négyzetösszege, aminek a szélsőértéke megkapható egy egyszerű deriválással.

Előzmény: [1154] farkasb, 2008-12-05 16:47:37
[1154] farkasb2008-12-05 16:47:37

Kedves Fórumozók!

Adott A(2,9,4) és B(6,7,1) ponttal meghatározott térbeli egyenes. Adott P(8,5,3) pont. Hogyan számolható ki a P pont vetülete az egynesen? Előre is köszönettel: farkasb

[1153] HoA2008-11-30 19:48:12

[1150] ábrájának tétele és a 142. feladat egyenértékűek. Jelöljük a D és E közötti piros pontot P-vel. A BE, CE, AD, BD egyenesek által meghatározott négy háromszög közül kettőnek a körülírt köre a P-n és E-n illetve a P-n és D-n áthaladó szaggatott körvonal. P tehát a négy háromszög körülírt köreinek közös pontja, a négy egyenes mint érintők által meghatározott parabola fókusza. A tétel bizonyítása tehát egyben a 142. feladat megoldása is.

Elemi, csak a húrnégyszögek szemközti szögeinek tulajdonságát felhasználó, lépésenként illusztrált bizonyítás található itt: http://agutie.homestead.com/FiLEs/miquel_pentagram1.htm

Előzmény: [1150] BohnerGéza, 2008-11-24 10:16:28
[1150] BohnerGéza2008-11-24 10:16:28

A 138. feladathoz kapcsolódik:

A "Valaki mondja meg!" témában az [573]. hozzászólásban ábra is mutatja a következő tételt:

Ha négy egyenes négy háromszöget alkot, akkor ezek magasságpontjai egy egyenesen vannak, körülírt köreik egy ponton mennek át. Ez a pont és a magasságpontok egyenese parabolát határoz meg, melynek az eredeti 4 egyenes érintője.

Nem ismerek elemi megoldást a következő 142. feladatra:

Bizonyítandó, ha öt egyenes közül bármelyik négy négy háromszöget alkot, akkor az ezekhez a fenti tétel szerint tartozó fókuszok egy körön vannak.

A tétel egy érdekes következménye: Egy konvex ötszög minden +második csúcsát összekötve kapott öt háromszög körülírt köreinek "második" metszéspontjai egy körön vannak. Ábra! Segítségül: http://mathworld.wolfram.com/MiquelsPentagramTheorem.html

Előzmény: [1138] HoA, 2008-10-20 08:50:44
[1149] HoA2008-11-15 08:07:57

Én viszont teljesen egyetértek BohnerGézával! A [1146] -beli általános leírás és a [1148] -ban kifejtett következtetés együtt teljes mértékben kiváltják a konkrét példára végzett [1147] -beli levezetést. Elnézést kérek, hogy két lépés összevonását nem vettem észre [1146] végén.

Előzmény: [1148] BohnerGéza, 2008-11-15 01:02:26
[1148] BohnerGéza2008-11-15 01:02:26

Kivételesen nem értek egyet HoA-val, bár ez sem teljesen igaz, csak ezért írom a következőt: Megfogalmazásom túlzottan tömör, komoly végiggondolást igényel, de szerintem teljes.

Az [1146]-ban leírtakból és az [1145]-ben lévő ábrából kiderül, hogy két forgatás szorzata végeredményben két olyan tükrözéssel helyettesíthető, melynek első egyenese az első forgatás kp-ján és az eredményül kapott forgatás kp-ján, a második a második forgatás kp-ján és az eredményül kapott forgatás kp-ján megy át, szögük az eredményforgatás szögének fele.

Kérem mindenkitől, ne adja föl, ha elsőre nem értette, amit írtam! Akarattal teszem, gondolkodást várok! (Ez nem HoA-nak szólt, jogosan figyelmeztetett mostani túlzott tömörségemre.)

Előzmény: [1147] HoA, 2008-11-14 23:02:29
[1147] HoA2008-11-14 23:02:29

A leírtak egészen az utolsó mondatig rendben vannak. Ha az utolsó mondatot mint egy itt nem részletezett bizonyítás eredményét tekintjük, akkor az is elfogadható. De önmagában nem. Természetesen ha KM és KN merőlegesek, akkor a rájuk történő tükrözések eredője egy 180 fokos forgatás, vagyis egy K pontra vonatkozó tükrözés. De hát éppen ezt a merőlegességet kéne bizonyítani. A [1146] -ban leírt forgatáshelyettesítésből nem következik, hogy éppen MK és NK lenne az a két egyenes, melyeket t1 -nek és t4 -nek választhatunk. Szerintem a [1146] -beli gondolat feladatunk megoldására az alábbiak szerint használható fel:

Az M körüli \pi-\phi szögű forgatás helyettesíthető két, egymással (\pi-\phi)/2 szöget bezáró, t1ést2 egyenesre vett tükrözéssel. Válasszuk t2 -nek MN -t, és legyen t1 az ezzel -(\pi-\phi)/2 szöget bezáró, M-en áthaladó egyenes. Hasonlóan az N körüli \phi szögű forgatás helyettesíthető két, egymással \phi/2 szöget bezáró, t3ést4 egyenesre vett tükrözéssel. Válasszuk t3 -nak MN -t, és legyen t4 az ezzel \phi/2 szöget bezáró, N-en áthaladó egyenes. Legyen t1 és t4 metszéspontja L. A két forgatás eredője, vagyis a négy tükrözés eredője, mivel a t2 -re és t3 -ra vonatkozó tükrözés eredője azonosság, a t1 -re majd t4 -re végrehajtott tükrözés eredője.

Az MNL háromszögben az M-nél és N-nél lévő szögek összege 90o , ezért a háromszög L-nél derékszögű. Másrészt a tükrözések eredője egy L köröli 180o -os forgatás - L-re vonatkozó tükrözés - és mivel a forgatások eredője C-t D-be viszi, L = K. Ezzel már valóban bizonyítottuk, hogy MK és NK merőlegesek.

Előzmény: [1146] BohnerGéza, 2008-11-11 23:30:46
[1146] BohnerGéza2008-11-11 23:30:46

Két forgatás szorzatáról: Ábra az előző hozzászólásban.

És a 141. feladat egy megoldása.

Előzmény: [1144] HoA, 2008-11-10 11:43:00
[1145] BohnerGéza2008-11-11 23:26:36

Ábra a forgatások szorzata egy lehetséges tárgyalásához. A következő hozzászólásban ennek egy részesetét vizsgáljuk, azt ha az eredmény nem eltolás.

Előzmény: [1141] BohnerGéza, 2008-10-31 00:06:46
[1144] HoA2008-11-10 11:43:00

A 141.feladat megoldása: Tükrözzük M-et K-ra, a tükörkép legyen M' .

a) Vegyük fel BM meghosszabbításaként az MB' = BM szakaszt. Jelöljük a BND szöget \phi-vel. ABND és ABMC húrnégyszög, ezért BAC és B'MC szög is \phi. Az ívfelezés miatt NB = ND és MB = MC (= MB') , a tükrözés miatt M'D = MC. ND az NB szakasz elforgatottja \phi-vel, a DM' -vel egyállású MC a BM -mel egyállású MB' elforgatottja \phi-vel, ezért NBM és NDM' háromszögek egybevágóak, NM = NM' . NK az MM'N egyenlőszárú háromszög alaphoz tartozó súlyvonala, tehát NK merőleges MK-ra.

b) Felhasználjuk, hogy egy X pont körüli \alpha szögű és egy Y pont körüli \beta szögű elforgatás eredője egy Z pont körüli \alpha+\beta szögű elforgatás, speciálisan ha \alpha+\beta=\pi , akkor egy Z pontra vonatkozó tükrözés. CMB szög = \pi-\phi, az M körüli \pi-\phi szögű elforgatás C-t B-be viszi, az N körüli \phi szögű elforgatás B-t D-be viszi, a két elforgatás eredője C-t D-be viszi. Mivel a két elforgatás eredője egy pontra vonatkozó tükrözés, ez a pont éppen K, tehát a két forgatás eredője egy K-ra vett tükrözés. Alkalmazzuk a két elforgatást M-re. Az M körüli forgatás M-et helyben hagyja, az N körüli forgatás egy olyan M' pontba viszi, melyre NM = NM' . Ugyanakkor M' éppen az M pont K-ra vett tükörképe. NK az MM'N egyenlőszárú háromszög alaphoz tartozó súlyvonala, tehát NK merőleges MK-ra.

Előzmény: [1143] BohnerGéza, 2008-11-05 23:18:14
[1143] BohnerGéza2008-11-05 23:18:14

A "kis segítség" önnálló feladatnak biztosan jó, ajánlom mindenkinek!

A 141. feladatra két megoldást ismerek, azokhoz nem használtuk ezt a segítséget, talán egy harmadikhoz jó.

Az általam ismett megodások alapszava:

- - - egyikhez: "ha felező..., akkor tükrözzünk rá!"

- - - másikhoz: "leképezések"

Előzmény: [1142] HoA, 2008-11-05 11:24:39
[1142] HoA2008-11-05 11:24:39

Egy kis segítség(?) a 141. feladathoz: Ha A és K egybeesik, akkor az MKN pontokon áthaladó kör érinti CD-t.

Előzmény: [1141] BohnerGéza, 2008-10-31 00:06:46

  [1]    [2]    [3]    [4]    [5]    [6]    [7]    [8]    [9]    [10]